You are on page 1of 154

Adhamjons SC notes

UZBEKISTAN

Note: These are my SC notes collected from testmagic.com. The explanations are those of 800Bob, a
Gmat instructor, who has got a perfect score. For new explanations, search on testmagic Gmat
forum. The explanations are very informative and helpful. I learned a lot from them. I thank
800Bob for such awesome explanations.
I hope my notes will help others.
All the best.

For the farmer who takes care to keep them cool, providing them with high-energy feed,
and milking them regularly, Holstein cows are producing an average of 2,275 gallons of
milk each per year.
A. providing them with high-energy feed, and milking them regularly, Holstein cows
are producing
B. providing them with high-energy feed, and milked regularly, the Holstein cow
produces
C. provided with high-energy feed, and milking them regularly, Holstein cows are
producing
D. provided with high-energy feed, and milked regularly, the Holstein cow produces
E. provided with high-energy feed, and milked regularly, Holstein cows will produce

Bobs EXPLANATION:

B and D are easily eliminated because "them" does not agree with "cow".

A has a number of problems:


1. The modifying phrases "providing them with high-energy feed" and "milking them regularly" are
located between two clauses. It is not clear whether they modify the subject "who" (= "farmers") of the
preceding clause or the subject "cows" of the following clause.
2. The comma between the two modifying phrases should be omitted.
3. The use of the progressive tense "are producing" is inappropriate to describe a general scientific truth.

C is wrong because "provided with high-energy feed" and "milking them regularly" are not parallel and
1

Adhamjons SC notes

UZBEKISTAN

because, like A, it uses the inappropriate progressive form "are producing".

E is perfect. The three adjectives "cool", "provided", and "milked" are parallel, short for: "to keep them
cool, keep them provided with high-energy feed, and keep them milked regularly".

And the future form "will produce" is appropriate for a general scientific truth.

A mixture of poems and short fiction, Jean Toomers Cane has been called one of the three best
novels ever written by Black Americansthe others being Richard Wright, author of Native Son,
and Ralph Ellison, author of Invisible Man.
A. Black Americansthe others being Richard Wright, author of Native Son, and Ralph Ellison,
author of Invisible Man

B. Black Americansincluding
Native Son by Richard Wright and Invisible Man by Ralph Ellison
C. a Black Americanincluding Richard Wright, author of Native Son, and Ralph Ellison, author
of Invisible Man

D. a Black Americanthe others being Richard Wright, author of


Native Son, and Ralph Ellison, author of Invisible Man

E. a Black Americanthe others being Richard Wrights


Native Son and Ralph Ellisons Invisible Man

Bobs explanation:

The best response is E.

The only potential problem with E is "the three best novels ever written by a Black American". But it is
not wrong here to use the singular "Black American" because each of the three novels was written by just
one person. Here are a couple of perfectly correct sentences that do something similar:
2

Adhamjons SC notes

UZBEKISTAN

According to the American Dietetic Association, children who eat a healthy breakfast are more likely to
have better concentration, problem-solving skills and eye-hand coordination.

Those with a college degree had almost twice as much income as those with just a high school diploma.

In the first example, each child eats just one breakfast. In the second, each person has just one college
degree and just one high school diploma.

Here are two correct sentences taken from Official Guides:

Three out of four automobile owners in the United States also own a bicycle. (OG10, SC 252)

Neanderthals had a vocal tract resembling an apes and so were probably without language, a
shortcoming that may explain why they were supplanted by our own species. (OG Verbal, SC 59)

For the first of these two examples there are a couple of answer choices that change "a bicycle" to
"bicycles", but as the official explanation says, "the plural bicycles detracts from clarity by suggesting that
multiple bicycles are owned by each person in question." Similarly, for the second of these two examples
there are three wrong answer choices that change "a vocal tract" to "vocal tracts".

So, in the example in question, choice B is wrong because it suggests that these novels each had multiple
authors. It is also wrong because "including" is imprecise, suggesting that Native Son and Invisible Man
are Black Americans.

You cannot eliminate E just because it includes the word "being". It is true that "being" is often wrong or
unnecessary, but to say that it is banned by the GMAT is ridiculous.

Climatic shifts are so gradual as to be indistinguishable at first from ordinary fluctuations in the
weather.
(A) so gradual as to be indistinguishable
(B) so gradual they can be indistinguishable
3

Adhamjons SC notes

UZBEKISTAN

(C) so gradual that they are unable to be distinguished


(D) gradual enough not to be distinguishable
(E) gradual enough so that one cannot distinguish them

Bobs explanation:

Use of the passive construction after able to or unable to is discouraged. It is not just awkward, but
illogical. To be distinguished from ordinary fluctuations is not an ability that climatic shifts can possess or
lack.
To take a simpler example, look at this ugly sentence:
This math problem is unable to be solved.
Better:
This math problem cannot be solved.

In no other historical sighting did Halley 's comet cause such a worldwide sensation as did its return in
1910-1911.

A did its return in 1910-1911


B had its 1910-1911 return
C in its return of 1910-1911
D its return of 1910-1911 did
E its return in 1910-1911

Bobs explanation:
If you begin the sentence this way, you can see more easily how to finish it:
Halley's comet caused such a worldwide sensation in no other historical sighting as ...
The comparison is between other historical sightings and that of 1910-1911, so you could finish the
sentence this way:
Halley's comet caused such a worldwide sensation in no other historical sighting as it did in its return of
1910-1911.
Deleting the superfluous words "it did" you get:
4

Adhamjons SC notes

UZBEKISTAN

Halley's comet caused such a worldwide sensation in no other historical sighting as in its return of 19101911.

So it is perfectly correct to write:

In no other historical sighting did Halley's comet cause such a worldwide sensation as in its return of
1910-1911.

The correct response is C.

According to a recent study of consumer spending on prescription medications, increases in the


sales of the 50 drugs that were advertised most heavily accounts for almost half of the $20.8 billion
increase in drug spending last year, the remainder of which came from sales of the 9,850
prescription medicines that companies did not advertise or advertised
very little.

A. heavily accounts for almost half of the $20.8 billion increase in drug spending last year, the
remainder of which came
B. heavily were what accounted for almost half of the $20.8 billion increase in drug spending last
year; the remainder of the increase coming
C. heavily accounted for almost half of the $20.8 billion increase in drug spending last year, the
remainder of the increase coming
D. heavily, accounting for almost half of the $20.8 billion increase in drug spending last year, while
the remainder of the increase came
E. heavily, which accounted for almost half of the $20.8 billion increase in drug spending last year,
with the remainder of it coming

Bobs explanation:

I wish to respectfully disagree with a couple of the points made above.

1. "In GMAT we look for the best and not the right answer."
5

Adhamjons SC notes

UZBEKISTAN

The idea that sometimes the 5 choices are all bad and that we are to identify the least bad choice is a myth.
There is never anything wrong with with correct response. The directions say to choose the "best" answer,
not the "correct" answer, because there can be more than one correct way to rephrase the sentence, but
only one of the correct ways to do so will be found among the answer choices.

2. "B changes the meaning of the original sentence."


There is nothing in the directions to Sentence Correction about finding a choice that does not change the
meaning of the original sentence. Indeed, sometimes it is necessary to change the meaning of the original,
that is, when the original is illogical. Here, for example, is the orignal sentence of OG number 95:

Unlike Schoenberg's 12-tone system that dominated the music of the postwar period, Bartk founded no
school and left behind only a handful of disciples.

In this form the sentence states that Schoenberg's 12-tone system was unlike Bartk. That's true, but it is
obviously not what the writer intended. What matters is not the literal meaning of the original sentence,
but the intended meaning. The objective is to find a choice that does not distort the intended meaning. The
challenge sometimes is to determine from a poorly written original what the writer really intended.

3. "...there was a run on."


Choice C is not a run-on sentence. A run-on is two independent clauses treated as a single sentence. Here
what follows the last comma is:

the remainder of the increase coming from sales of the 9,850 prescription medicines that companies did
not advertise or advertised

That's not an independent clause. It could not stand alone: there's no conjugated verb. If you really want to
know what grammatical form we have here... It's a nominative absolute. Check out Nominative absolute Wikipedia, the free encyclopedia.

There is nothing wrong with C, and so C is the best response.

Over 75 percent of the energy produced in France derives from nuclear power, while in Germany it
is just over 33 percent.

Adhamjons SC notes

UZBEKISTAN

A. while in Germany it is just over 33 percent


B. compared to Germany, which uses just over 33 percent
C. whereas nuclear power accounts for just over 33 percent of the energy produced in Germany
D. whereas just over 33 percent of the energy comes from nuclear power in Germany
E. compared with the energy from nuclear power in Germany, where it is just over 33 percent

Bobs explanation:
A is wrong because it refers illogically to the subject of the preceding clause 75 percent of the energy
produced in France. It makes no sense to say that 75 percent of the energy produced in France is just over
33 percent in Germany.
B is wrong because it compares 75 percent of the energy produced in France with Germany.
D is wrong because it says that 33 percent of the energy produced in France comes from nuclear power in
Germany. How can energy produced in France come from Germany?
E is wrong because it is awkward and confusing, and specifically because the reference of the pronoun it
is unclear. What is just over 33 percent? If it's nuclear power that's just over 33 percent, then just over 33
percent of what?

The correct response is C

Minivans carry as many as seven passengers and, compared with most sport utility vehicles, cost
less, get better gas mileage, allow passengers to get in and out more easily, and have a smoother
ride.

A.Minivans carry as many as seven passengers and, compared with most sport utility vehicles, cost
less,
B.Minivans, which carry as many as seven passengers, compared with most sport utility vehicles,
they cost less,
C.Minivans carry as many as seven passengers, in comparison with most sport utility vehicles, and
have a lower cost, they
D.Minivans, carrying as many as seven passengers, compared with most sport utility vehicles, cost
less,

Adhamjons SC notes

UZBEKISTAN

E.Minivans, which carry as many as seven passengers, compared with most sport utility vehicles the
cost is lower, and they

Bobs explanation:

D is awkward and confusing in that it has two modifying phrases ("carrying as many as seven passengers"
and "compared with most sport utility vehicles") between the subject "minivans" and the first verb "cost".

Reading the sentence as choice D phrases it, when I get to "compared" I first think it's the verb. Only
when I get to "cost" do I realize that the phrase beginning with "compared" is not the predicate but a
second modifier. D is confusing also in that it is not clear in what way minivans are being compared with
most SUVs. Is the fact that minivans carry as many as seven passengers one of the differences?

There is absolutely nothing wrong with choice A. It tells us two things about minivans:
Minivans carry as many as seven passengers.
Minivans (compared with most SUVs) cost less, etc.
Since these two facts have the same subject, we can put them together, without repeating the subject, and
inserting "and" between the verbs:
Minivans carry as many as seven passengers and (compared with most SUVs) cost less, etc.
It would be wrong to put a comma in front of "and". On the other hand, the comma after "and" makes
perfect sense: it's one of a pair of commas setting off the modifying phrase "compared with most SUVs".

The fact of some fraternal twins resembling each other greatly and others looking quite dissimilar
highlights an interesting and often overlooked feature of fraternal-twin pairs, namely they vary
considerably on a spectrum of genetic relatedness.

A. The fact of some fraternal twins resembling each other greatly and others looking quite
dissimilar highlights an interesting and often overlooked feature of fraternal-twin pairs, namely
they vary considerably

B. That some fraternal twins resemble each other greatly while others look quite dissimilar
highlights an interesting and often overlooked feature of fraternal-twin pairs, namely that they vary
8

Adhamjons SC notes

UZBEKISTAN

considerably

C. With some fraternal twins resembling each other greatly and others looking quite dissimilar, it
highlights an interesting and often overlooked feature of fraternal-twin pairs, namely considerable
variation

D. With some fraternal twins resembling each other greatly and others looking quite dissimilar, it is
a fact that highlights an interesting and often overlooked feature of fraternal-twin pairs, namely a
considerable variation

E. Because some fraternal twins resemble each other greatly and others look quite dissimilar, this
fact highlights an interesting and often overlooked feature of fraternal-twin pairs, namely they vary
considerably

EXPLANATION BY BOB:

This is a case in which the subject is a whole clause: "that some fraternal twins resemble each other
greatly while others look quite dissimilar." That is the thing that is doing the highlighting.

A IS WRONG:
The fact of + noun + gerund..." - unidiomatic.
"The fact that + noun + verb..." - better.
"That + noun + verb...." - best.

Example:
"The fact of the GMAT being hard cannot be denied." - unidiomatic and ugly.
"The fact that the GMAT is hard cannot be denied." - OK, but wordy.
"That the GMAT is hard cannot be denied." - best

Malaria ravages more people than any disease, yet there are only a dozen laboratories in the world
which are devoted to its study.
(A) than any disease, yet there are only a dozen laboratories in the world which
(B) than any disease, but there are only a dozen laboratories in the world which
9

Adhamjons SC notes

UZBEKISTAN

(C) than does any disease, but only a dozen laboratories in the world
(D) as any other disease, but only a dozen laboratories in the world
(E) than any other disease, yet only a dozen laboratories in the world

EXPLANATION BY BOB:

in this case the word "other" is enough to make it clear that the point of comparison is "malaria." In
correct choice E:

"Malaria ravages more people than any other disease..."

there can be no ambiguity. "People" are not a disease. It would not be wrong to write:

"Malaria ravages more people than does any other disease..."

but it is wrong to omit "other."

Leaching, the recovery of copper from the drainage water of mines, as a method of the
extraction of minerals, it was well established as early as the eighteenth century, but until
about 25 years ago miners did not realize that bacteria take an active part in the process.
A. as a method of the extraction of minerals, it was well established
B. as a method of the extraction of minerals well established
C. was a well-established method of mineral extraction
D. was a well-established method of extracting mineral that was
E. had been a method of mineral extraction, well established
EXPLANATION BY BOB:

There is no way the answer to number 1 can be A. It contains a double subject: "Leaching... it was..."

In number 1, the past perfect is not necessary. The temporal relationship is made perfectly clear by the
words "but until about 25 years ago." Another example: "Until the 1960s New York was the most
10

Adhamjons SC notes

UZBEKISTAN

populous state in the Union, but in the 1970s and 80s the state began to experience an economic decline."
No need to write: "had been the most populous state."

In any case, the past perfect makes no sense the way it is used in E in that it suggests that leaching is no
longer a method of mineral extraction. Past perfect would not be wrong, but it would have to be written
something like: "had been a well-established method of mineral extraction."

The answer is C.

According to United States census data, while there was about one-third of mothers
with young children working outside the home in 1975, in 2000, almost two-thirds of
those mothers were employed outside the home.

A.while there was about one-third of mothers with young children working outside
the home in 1975, in 2000, almost two-thirds of those mothers were employed outside
the home.

B.there were about one-third of mothers with young children who worked outside the
home in 1975; in 2000, almost two-thirds of those mothers were employed outside the
home

in 1975 about one-third of mothers with young children worked outside the home; in
2000, almost two-thirds of such mothers were employed outside the home

even though in 1975 there were about one-third of mothers with young children who
worked outside the home, almost two-thirds of such mothers were employed outside
the home in 2000

with about one-third of mothers with young children working outside the home in
1975, almost two-thirds of such mothers were employed outside the home in 2000
11

Adhamjons SC notes

UZBEKISTAN

OA is C.

800BOB:
a variety of"
"a number of"
"a total of"
...plural

"the variety of"


"the number of"
"the total of"
...singular

A variety of ideas were presented.


The variety of ideas was mind-boggling.

BOB:

Here are three correct sentences from 1000 SC's in which "that" refers to a noun earlier than the
immediately preceding one:

Out of Americas fascination with all things antique has grown a market for bygone styles of furniture and
fixtures that is bringing back the chaise lounge, the overstuffed sofa, and the claw-footed bathtub.

Dr. Sayres lecture recounted several little-known episodes in the relations between nations that illustrate
what is wrong with alliances and treaties that do not have popular support.

In good years, the patchwork of green fields that surrounds the San Joaquin Valley town bustles with
farm workers, many of whom are in the area just for the season.

12

Adhamjons SC notes

UZBEKISTAN

EXPLANATION BY BOB:

Relative pronoun (that, who, which) agree with the immediately preceding noun.

Subject and object pronouns (it, they, them) agree with the subject of the preceding clause or sentence

I asked someone who is far far better than I am at determining the correct rules and grammar in general
what his thoughts were on this subject and he (he's someone who had a perfect score on the exam) said,
"In general, a participial phrase set off from the rest of the sentence with commas is understood to modify
the subject of the sentence"

It is misleading and oversimplistic to say that "which" must always be preceded by a comma. The rule is
that a nonrestrictive clause should be set off with commas, but a nonrestrictive clause does not always
begin with "which," and the presence of "which" does not always signal a nonrestrictive clause.

First of all, if the relative pronoun is the object of a preposition, whether the clause is restrictive or
nonrestrictive, you cannot use "that"; you must use "which." Here are a couple of correct sentences from
the OG that use "which" after a preposition in a restrictive clause:

El Nino, the periodic abnormal warming of the sea surface off Peru, is a phenomenon in which changes in
the ocean and atmosphere combine to allow the warm water that has accumulated in the western Pacific
to flow back to the east.

As U.S. nuclear attack submarines prowl their familiar haunts deep within the oceans of the world these
days, they increasingly are engaged in missions far different from the tasks for which they were built and
for which their crews were trained over the last forty years

Chicago, where industrial growth in the nineteenth century was more rapid than any other
American city, was plauged by labour troubles like the Pullman Strikes of 1894.
A) where industrial growth in the nineteenth century was more rapid than any other American city
13

Adhamjons SC notes

UZBEKISTAN

B) which had industrial growth in the nineteenth century more rapid than that of other American
cities
C) which had growth industurally more rapid than any other American city in the ninteenth
century
D) whose industrial growth in the nineteenth century was more rapid than any other American city
E) whose industrial growth in the nineteenth century was more rapid than of any
other American city

EXPLANATION BY BOB:

Something's wrong here. All choices are wrong. Most other choices have comparison problems.

A: says that Chicago's growth was "more rapid than any other American city." Needs to be "more rapid
than that of any other American city."

B: "had industrial growth... more rapid than that of other American cities." Here "that of" makes no sense.
If you're going to start the comparison with "had," you need to finish with a verb, as in "than other
American cities had."

C: Adverb "industrially" in "had growth industrially more rapid" seems to modify "more rapid" instead of
"had growth."

D: like A, needs "that of."

E: Maybe this is supposed to be the correct answer and it's just mistyped. I see "of" where I want to see
"that of."

You can say either:


which had industrial growth more rapid than other American cities had
or:
whose industrial growth was more rapid than that of other American cities
But B confuses the two. "That of other American cities" needs to be compared with some sort of
possessive form, such as "the industrial growth of which" or "whose industrial growth."

14

Adhamjons SC notes

UZBEKISTAN

Maybe an analogous but simpler example will help. You can say either:
The area of Russia is greater than that of India.
or:
Russia has a greater area than India has.
But it's imprecise to say:
Russia has a greater area than that of India.

BOB:

before Australia was Australia, it was the antipodes MEANS


It means: before the country that is now know as Australia got that name.

This is not such a strange expression. Try googling on "before America was America" or "before England
was England" and you'll get hundreds of hits.

Although all the proceedings of the Communist party conference held in Moscow were not carried

live, Soviet audiences have seen a great deal of coverage.

(A) all the proceedings of the Communist party conference held in Moscow were not carried live

(B) all the Communist party conference's Moscow proceedings were not carried live

(C) all the Communist party conference Moscow proceedings have not been carried alive

(D) not all the Communist party conference Moscow proceedings have been carried alive

(E) not all the proceedings of the Communist party conference held in Moscow were carried live

What is wrong with th given sentence? The OA is E

15

Adhamjons SC notes

UZBEKISTAN

EXPLANATION BY BOB:

A and E do not mean the same thing.

A, which says that "all the proceedings were not carried live," means that no proceeding was carried live.
Though not grammatically incorrect, A is awkward and clearly not what the writer intended, since
"audiences have seen a great deal of coverage."

There is a difference between "All X are not Y" and "Not all X are Y." Consider the following:

1. All of us are not going.


2. Not all of us are going.

In the first example, no one is going. In the second, some, but not all, are going.

The construction used in A is a mistake commonly made by native speakers. Here are a few examples I
found on the Internet:

1. "Althen is quick to acknowledge that all Americans are not white and middle-class."
Wrong. Should be: "...not all Americans are white and middle-class," or "...Americans are not all white
and middle-class."

2. "All prime numbers are not odd."


Wrong. Should be: "Not all prime numbers are odd", or "Prime numbers are not all odd."

3. "All languages are not based on the English alphabet."


Wrong. Should be: "Not all languages are based on the English alphabet," or "Languages are not all based
on the English alphabet."

In a period of time when women typically have had a narrow range of choices, Mary Baker Eddy
became a distinguished writer and the founder, architect, and builder of a growing church.
16

Adhamjons SC notes

UZBEKISTAN

(A) In a period of time when women typically have


(B) During a time in which typically women have
(C) Typically, during a time when women
(D) At a time when women typically
(E) Typically in a time in which women

EXPLANATION BY BOB:

and B are wordy, but the real reason to eliminate them is "have." The present perfect is wrong when
describing an event that happened and was completed in the past. C and E are wrong because "typically"
is misplaced.

Quote:
Originally Posted by zombie_in
As per rules whenever we use "which" it should have comma(,) in front of it

Not true. You don't put a comma in front of "which" when it follows a preposition. "In which" is perfectly
fine.

California remains the state in which most Americans wish they could live.
A historical novel is a novel in which the story is set among historical events.

Would you really put a comma in front of which?

California remains the state in, which most Americans wish they could live. (sic)
A historical novel is a novel in, which the story is set among historical events. (sic

Believed to originate from a small area on their foreheads, elephants emit low-frequency sounds
that may be used as a secret language to communicate with other members of the herd.
(A) Believed to originate from a small area on their foreheads, elephants emit low-frequency sounds
that may be used
17

Adhamjons SC notes

UZBEKISTAN

(B) Elephants emit low-frequency sounds that are believed to originate from a small area on their
foreheads, and they may use this
(C) Elephants emit low-frequency sounds, believed to originate from a small area on their
foreheads, that they may use
(D) Originating, it is believed, from a small area on their foreheads; elephants emit low-frequency
sounds they may use
(E) Originating, it is believed, from a small area on their foreheads, low-frequency sounds are
emitted by elephants that may be used

isnt 'they' in choice B refers to subject 'elephant' ........

EXPLANATION BY BOB:

Problem with B is "this," which is supposed to refer to "sounds."

Best response is C.

Sunspots, vortices of gas associated with strong electro-magnetic activity, are visible as dark spots
on the surface of the Sun but have never been sighted on the Sun's poles or equator.
(A) are visible as dark spots on the surface of the Sun but have never been sighted on
(B) are visible as dark spots that never have been sighted on the surface of the Sun
(C) appear on the surface of the Sun as dark spots although never sighted at
(D) appear as dark spots on the surface of the Sun, although never having been sighted at
(E) appear as dark spots on the Suns surface, which have never been sighted on

EXPLANATION:

If you want "although never sighted" to modify "sunspots," you would need to move it to the beginning of
the sentence. Placed at the end, "although" needs to be followed by a subject and conjugated verb. "At"
versus "on" is a red herring -- they're both OK.

It is counterproductive to disagree with the OG. The people who write the test say the answer is A. It's not
18

Adhamjons SC notes

UZBEKISTAN

a typographical error. They offer a detailed explanation for why A is right and C is wrong. If one wants to
get a high score on the GMAT, one needs to learn to think the way the test makers think.

In the most common procedure for harvesting forage crops such as alfalfa, as much as 20 percent of
the leaf and small-stem material, which is the most nutritious of all the parts of the plant, shattered
and fell to the ground.
(A) which is the most nutritious of all the parts of the plant, shattered and fell
(B) the most nutritious of all parts of the plant, shatter and fall
(C) the parts of the plant which were most nutritious, will shatter and fall
(D) the most nutritious parts of the plant, shatters and falls
(E) parts of the plant which are the most nutritious, have shattered and fallen
OA D

EXPLANATION BY BOB:

There are two ways to look at the subject "20 percent of the leaf and small-stem material."

One way is to think it means 20 percent of two different things: "leaf" and "small-stem material." This
way doesn't work because you can't say "20 percent of the leaf." "Leaf" is a countable noun. It would have
to be "20 percent of the leaves."

The other way is to think it means 20 percent of the material. The material consists of leaves and small
stems, and so it can be called "leaf and small-stem material." This way works.

So for me, the subject is singular. The present tense also works for me. So I would choose D. But I have a
problem with D. If "leaf and small-stem material" is singular, then I think it would be better for the
following appositive to be singular also: "the most nutritious part of the plant."

All in all, not a good question. Clearly not GMAT-quality.

19

Adhamjons SC notes

UZBEKISTAN
Pronoun 'they'

1. While depressed property values can hurt some large investors, they are
potentially devastating for home-owners, whose equityin many cases
representing a lifes savingscan plunge or even disappear.

(A) they are potentially devastating for homeowners, whose

(B) they can potentially devastate homeowners in that their

(C) for homeowners they are potentially devastating, because their

(D) for homeowners, it is potentially devastating in that theirA

(E) it can potentially devastate homeowners, whose

Why is (A) correct? the use of 'they' can either represent 'proporty values' or
'large investors'. Please let me know.

EXPLANATION BY BOB:

Plural pronouns do not necessarily refer to the nearest plural noun, and singular pronouns do not
necessarily refer to the nearest singular noun. If that were true, there would be no such thing as ambiguous
pronoun reference. In the OG alone there are dozens of SC items in which the correct answer choice
includes a pronoun whose antecedent is not the nearest available noun. In addition to the item quoted
above (SC 159), here are just six more:

97. Iguanas have been an important food source in Latin America since prehistoric times, and they are
still prized as game animals by the campesinos....
The pronoun they refers to iguanas, not to the nearest plural noun times.

20

Adhamjons SC notes

UZBEKISTAN

103. Students in the metropolitan school district are so lacking in math skills that it will be difficult to
absorb them into a city economy....
The pronoun them refers to students, not to the nearest plural noun skills.

152. ...a microbe never before seen on Earth that might escape from the laboratory and kill vast numbers
of humans who would have no natural defenses against it.
The pronoun it refers to microbe, not to the nearest singular noun laboratory.

153. A recording system was so secretly installed and operated in the Kennedy Oval Office that even
Theodore C. Sorensen, the White House counsel, did not know it existed.
The pronoun it refers to system, not to the nearest singular noun Office.

180. Quasars are so distant that their light has taken billions of years to reach the Earth; consequently,
we see them as they were during the formation of the universe.
The pronouns them and they refer to quasars, not to the nearest plural noun years.

251. The gyrfalcon, an Arctic bird of prey, has survived a close brush with extinction; its numbers are now
five times greater than when....
The pronoun its refers to gyrfalcon, not to the nearest singular noun extinction.

The rule is not so simply stated as "a pronoun will refer to the nearest available noun." In fact, as all the
above examples show, if a pronoun agrees in number with the subject of the preceding clause, then that
subject will normally be the antecedent, even if there is another noun of the same number closer to the
pronoun.

According to GMAT which is correct


This is me. or This is I.

EXPLANATION BY BOB:

I can't recall ever seeing either the word "me" or the word "I" in a GMAT Sentence Correction item.

21

Adhamjons SC notes

UZBEKISTAN

Strictly speaking, you should use the subject pronoun after forms of the verb "be."

It is I.
It was they who arrived first.
"Hello. May I speak to Bob?" "This is he."
If you were I...

This is because "be" is not a transitive verb. A pronoun after "be" is a complement, not a direct or indirect
object, and so grammatically speaking it is incorrect to use an object pronoun.

But only grammar pedants like me speak this way. Normal people say, "It's me!"

In any case, don't worry. This is never tested on the GMAT.

In large doses, analgesics that work in the brain as antagonists to certain chemicals have caused
psychological disturbances in patients, which may limit their potential to relieve severe pain.
(A) which may limit their potential to relieve
(B) which may limit their potential for relieving
(C) which may limit such analgesics potential to relieve
(D) an effect that may limit their potential to relieve
(E) an effect that may limit the potential of such analgesics for relieving

I have a question on this problem, I was able to get till D & E and then picked D thinking that 'their' refers
to analgesics because as told by Bob, pronoun refers to subject of previous clause, which is "analgesics" .
Also E looks to me lengthy
plz advise where i'm going wrong...thanx!!

EXPLANATION BY BOB:

I say E.

22

Adhamjons SC notes

UZBEKISTAN

This is an interesting item. I'm not sure it's a real GMAT item, but I still think it's a useful one for
illustrating the dangers of following a rule of grammar too literally. Those of us with extremely analytical
minds would like to think that the rules of grammar can be expressed with precision. We must face the
fact, however, that language is not strictly logical.

I stand by the guideline that a pronoun will tend to refer to the subject of the preceding phrase if the
pronoun and subject agree in number. (We have seen many, many examples where the pronoun does not
automatically refer to the nearest noun.) So in this case it would seem that "their" in choice D refers
grammatically and logically to "analgesics." The problem with D, however, is that there are just too many
other plural nouns between "analgesics" and "their": "antagonists," "chemicals," "disturbances,"
"patients." For this reason it is better to write "of such analgesics" than "their."

Here's what I tell my students: If you have a choice that uses a pronoun and a choice repeats the noun,
generally you should go for the one that repeats the noun. A little repetition for the sake of clarity is not a
bad thing.

Classical guitar was neither prestigious nor was often played in concert halls until it was revived by
Andres Segovia in the mid-twentieth century, having been won over by the instrument's sound
despite its relative obscurity.

A. Classical guitar was neither prestigious nor was often played in concert halls until it was revived
by Andres Segovia in the mid-twentieth century, having been won over by the instrument's sound
despite its relative obscurity.
B. Classical guitar was neither prestigious nor played often in concert halls until it was revived by
Andres Segovia in the mid-twentieth century, having been won over by the instrument's sound
despite its relative obscurity.
C. Classical guitar was not prestigious and was not often played in concert halls until Andres
Segovia revived it in the mid-twentieth century, after he was won over by the sound despite the
instrument's relative obscurity.
D. Classical guitar did not have prestige nor was it performed often in concert halls until its revival
by Andres Segovia, who in the mid-twentieth century was won over by the instrument's sound
despite its relative obscurity.
23

Adhamjons SC notes

UZBEKISTAN

E. Classical guitar was neither prestigious nor was often played in concert halls until Andres
Segovia revived it in the mid-twentieth century, when he was won over by the sound of the relatively
obscure instrument.
OA C

EXPLANATION BY BOB:

For me the best choice is C.

A and E are wrong because "neither prestigious nor was often played" is not parallel.

A and B are wrong because the participial phrase "having been won..." seems to modify (illogically) the
subject of the sentence, "classical guitar."

D is wrong because "played" is changed to "performed." You don't perform an instrument; you play it or
perform on it. D also has an ambiguous "its."

I see nothing wrong with C.

With its plan to develop seven and a half acres of shore land, Cleveland is but one of a large number
of communities on the Great Lakes that is looking to its waterfront as a way to improve the quality
of urban life and attract new businesses.

(A) is looking to its waterfront as a way to improve the quality of urban life and attract

(B) is looking at its waterfront to improve the quality of urban life and attract

(C) are looking to their waterfronts to improve the quality of urban life and attract

(D) are looking to its waterfront as a way of improving the quality of urban life and attracting

24

Adhamjons SC notes

UZBEKISTAN

(E) are looking at their waterfronts as a way they can improve the quality of urban life and attract

OA C

EXPLANATION BY BOB:

The verb must be "are": Cleveland is one of many communities that are... "That" refers to "communities"
and is therefore plural.

Note the difference between "one of the [plural noun] is" and "one of the [plural noun] that are":

One of the most powerful driving forces behind recycling is the threat of legislation that would require
companies to take more responsibility for the disposal of their products.

Twenty-two feet long and 10 feet in diametier, the AM-1 is one of the many new satellites that are part of
a 15-year effort to subject the interactions of Earth's atmosphere, oceans, and land surfaces to detailed
scrutiny from space.

Last edited by 800Bob : 07-08-2005 at 06:20 PM.


Looking at: Physically act of seeing sthg/someone
Looking to: Depending on, turning to, expecting sthg from..
Look to the waterfront to improve my business.. I have started considering that waterways are a good
means to improve business..
Look up to you - Turn to you for advice..

Are they both acceptable?

Mr. King agrees it is possible that planets orbiting .... could ...

Mr. King agrees that it is possible for planets orbiting ... to ....

Any ideas?

25

Adhamjons SC notes

UZBEKISTAN

EXPLANATION BY BOB:

"It is possible that planets could" is redundant. It's better not to use "could," "might," or "may" with
"possible" or "possibly."

Bob, is it okay to say

It is possible for him to do something

I am not sure about this ''for x to do'' usage. What do u say?

Yes, it is possible for you to say that.

A 1972 agreement between Canada and the United States reduced the amount of phosphates that
municipalities had been allowed to dump into the Great Lakes.

(A) reduced the amount of phosphates that municipalities had been allowed to dump
(B) reduced the phosphate amount that municipalities had been dumping
(C) reduces the phosphate amount municipalities have been allowed to dump
(D) reduced the amount of phosphates that municipalities are allowed to dump
(E) reduces the amount of phosphates allowed for dumping by municipalities

EXPLANATION BY BOB:

agree with D. An agreement cannot reduce the amount that had been allowed. That previous amount
cannot be changed. There is an allowable amount. That amount was bigger before the agreement, and it is
smaller now as a result of the agreement.

Analogy: Let's say the legislature has changed the speed limit. Which sounds better?

26

Adhamjons SC notes

UZBEKISTAN

1) The legislature increased the maximum speed that one had been allowed to drive.
2) The legislature increased the maximum speed that one is allowed to drive.

I'll go with 2.
Past perfect is used to suggest that something is over and done with before the main action of the past. It
would be OK to say:

The 1972 agreement replaced the amount of phosphates that had been allowed with a new amount.

But if you are talking about changing an amount, then the amount existed before and continues to exist
after the 1972 agreement. The present tense is used to refer to something that exists for all time.

Copernicus revealed that the Earth and the planets all revolve around the Sun.

"Revolve" is in the present tense because it is an action that was then and continues to take place. It would
be wrong to say: "...revealed that the Earth and the planets revolved..." or "had revolved."

In the sentence under dicussion, there was and continues to be an amount of phosphates that municipalities
can dump. The 1972 agreement reduced the amount, but the amount continues to exist.

I'll try one more example. Suppose I started a new diet last week. I formerly ate all the red meat I wanted.
Under the new diet I allow myself to eat only 100 grams a day.

The diet I started last week reduced the amount of red meat I am allowed to eat to 100 grams a day.

It would be wrong to say:

The diet I started last week reduced the amount of red meat I had been allowed to eat to 100 grams a day.

GOOD INSIGHT
How can a agreement reduce something that has already been dumped?
Choice D is perfect.
A is ,initially, awfully tempting.
Brilliant Question!!
27

Adhamjons SC notes

UZBEKISTAN

17. Bob Wilber became Sidney Bechets student and protege when he was nineteen and, for a few
years in the 1940s, came as close to being a carbon copy of the jazz virtuoso in performance as
anyone has ever come.

(A) as anyone has ever come

(B) as anyone ever had been

(C) as anyone ever had done

(D) that anyone ever did

(E) that anyone ever

OA-C

EXPLANATION BY BOB:

The "OA" is wrong. The best answer is A.

B and C are both wrong because "ever" is misplaced. Adverbs of frequency ("always," "never," "often,"
"seldom," "ever," etc.) must be placed between the auxiliary verb and the past participle. Should be: "as
anyone had ever done."

There is nothing wrong with using the present perfect "has come" in A. It means that Wilber came as close
to being a carbon copy of the jazz virtuoso as anyone had come before or has come since.

Consider the following examples:

"In 1960 Johnson scored as many goals as anyone had ever scored in one year."
"In 1960 Johnson scored as many goals as anyone has ever scored in one year."

28

Adhamjons SC notes

UZBEKISTAN

In the first example, Johnson's number of goals in 1960 is compared only with previous years. In the
second example his number of goals in 1960 is compared with all years before and since.
Either past perfect or present perfect would be correct. But C puts "ever" in the wrong place and so cannot
be correct.

I see nothing wrong with A. For a few years in the 1940s Wilber came very close to being a carbon copy
of his teacher. To this day, no one has ever come closer.

I have seen quite a few cases in which the so-called "OA" is wrong.

1. A survey by the National Council of Churches showed that in 1986 there were 20,376 female
ministers, almost 9 percent of the nation's clergy, twice as much as 1977.
(A) twice as much as 1977
(B) twice asn many as 1977
(C) double what it was in 1977
(D) double the figure for 1977
(E) a number double that of 1977's
SPOILER: OA - D

2. According to surveys by the National Institute on Drug Abuse, about 20 percent of young adults
used cocaine in 1979, doubling those reported in the 1977 survey.

(A) doubling those reported in the 1977 survey

(B) to double the number the 1977 survey reported

(C) twice those the 1977 survey reported

(D) twice as much as those reported in the 1977 survey

(E) twice the number reported in the 1977 survey

29

Adhamjons SC notes

UZBEKISTAN

SPOILER: OA - E

EXPLANATION BY BOB:

Yes, the dictionary says that "twice" is an adverb, but it's perfectly idiomatic to say "twice the number."
Putting "twice" before "the number" does not make "twice" an adjective. If it were an adjective you'd have
to place it after "the," but you can't say "the twice number."

VERY GOOD POINT HERE:


In April 1997, Hillary Rodham Clinton hosted an all-day White House scientific conference on new
findings that indicates a childs acquiring language, thinking, and emotional skills as an active
process that may be largely completed before age three.

A. that indicates a childs acquiring language, thinking, and emotional skills as


B. that are indicative of a child acquiring language, thinking, and emotional skills as
C. to indicate that when a child acquires language, thinking, and emotional skills, that it is
D. indicating that a childs acquisition of language, thinking, and emotional skills is
E. indicative of a childs acquisition of language, thinking, and emotional skills as

GOOD EXPLANATION BY BOB:

I don't like E at all. "Indicative of" is idiomatically correct in some cases, but not here. "Indicative of"
should be followed by a noun.

Example: Some believe that facial characteristics are indicative of personality traits.

The noun "personality traits" is what is indicated.

Example: Are these averages indicative of what a typical family pays for medical care?

The noun phrase "what a typical family pays" is what is indicated.


30

Adhamjons SC notes

UZBEKISTAN

But to say: "indicative of a child's acquisition of ... skills as an active process" is awkward and imprecise.
It's not the skills that are indicated; it's the fact that these skills are an active process.

I see nothing wrong with D. The objection some have raised is that the participial phrase "indicating that a
child's acquisition..." might could be felt to modify the subject "Hillary Rodham Clinton" instead of the
immediately preceding noun "findings." For a participial phrase at the end of a sentence to modify a
distant subject instead of an adjacent noun, it must be separated from the rest of the sentence with a
comma, like this:

Hillary Rodham Clinton hosted an all-day White House scientific conference on new findings, indicating
that a childs acquisition of language, thinking, and emotional skills is an active process that may be
largely completed before age three.

Without a comma between "findings" and "indicating," choice D is not ambiguous

The health commissioner said that the government had implemented strict measures to eradicate
the contaminated food and, despite the recent illnesses, it will try to prevent the outbreak from
reccurring in the future.

A) it will try

B) that it tried

C) it had tried

D) it would try

E) that it would try

OA-E

EXPLANATION :
31

Adhamjons SC notes

UZBEKISTAN

"Would" is used to refer to the future from the point of view of the past. "He said he would arrive shortly."

The Health commissioner said X and Y ...

X == that the government had implemented strict measures to eradicate the contaminated food

Y= that the government would try ..

I hope this will make u clear smudge ...

A good Question . Testing Parallelism .. I was also wrong initiallly , Now i learned it.

Thanks for the question.

Employment costs rose 2.8% in the 12months that ended in sept,slightly(less than they did) in the
year that ended in the previous quarter.
A......
B.lower than
C.lower than they were
D.lower than they did
E.less than

OA-A

EXPLANATION BY BOB:

The sentence is not comparing employment costs of two different periods. It is comparing how much the
costs rose in two different periods. Employment costs rose by a smaller amount in the 12 months that
ended in September than they rose in the 12 months that ended in June. Employment costs rose 2.8% in
the more recent period, slightly less than they rose in the earlier period.

32

Adhamjons SC notes

UZBEKISTAN

There seem to be two different versions of this item in GMATPrep. One is the version imperfectly typed
at the top of this thread. That version actually looks like this:

In addition to her work on the Miocene hominid fossil, Mary Leakey contributed to archaeology
through her discovery of the earliest direct evidence of hominid activity and through her
painstaking documentation of East African cave paintings.

A) Mary Leakey contributed to archaeology through her discovery of the earliest direct evidence of
hominid activity and through her painstaking documentation of

B) Mary Leakey contributed to archaeology by her discovery of the earliest direct evidence of
hominid activity and painstakingly documenting

C) Mary Leakey was a contributor to archaeology by discovering the earliest direct evidence of
hominid activity and with her painstaking documentation of

D) Mary Leakey's contributions to archaeology include her discovery of the earliest direct evidence
of hominid activity and painstakingly documenting

E) Mary Leakey's contributions to archaeology include her discovering the earliest direct evidence
of hominid activity and painstaking documentation of

Correct response:
SPOILER: A

The other version looks like this:

In addition to her work on the Miocene hominid fossil record, Mary Leakey contributed to
archaeology with her discovery of the earliest direct evidence of hominid activity and painstakingly
documenting East African cave paintings.

A) Leakey contributed to archaeology with her discovery of the earliest direct evidence of hominid
33

Adhamjons SC notes

UZBEKISTAN

activity and painstakingly documenting

B) Leakey contributed to archaeology by her discovery of the earliest direct evidence of hominid
activity and by painstakingly documenting

C) Leakey was a contributor to archaeology with her discovery of the earliest direct evidence of
hominid activvity and with her painstaking documentation of

D) Leakey's contributions to archaeology include her discovery of the earliest direct evidence of
hominid activity and her painstaking documentation of

E) Leakey's contributions to archaeology include discovering the earliest direct evidence of hominid
activity and painstaking documentation of

Correct response:
SPOILER: D

Dr. Sayres lecture recounted several little-known episodes in the relations between nations that
illustrates what is wrong with alliances and treaties that do not have popular support.
(A) relations between nations that illustrates
(B) relation of one nation with another that illustrates
(C) relations between nations that illustrate
(D) relation of one nation with another and illustrate
(E) relations of nations that illustrates

EXPLANATION:
The correct response is C.

Subject-verb agreement problem in A, B, and E. Should be: "...several little-known episodes ... that
illustrate..."

"Between" is just fine here. From The Columbia Guide to Standard American English:

34

Adhamjons SC notes

UZBEKISTAN

"It is often argued that between should be used to express a relationship involving two of something, and
among should express relationships involving three or more, but in fact that generalization does not
describe the way English has long used these prepositions. Between can be used of as many items as you
like if the relationship is one-to-one, however much it may be repeated with different partners: Economic
relations between Great Britain, France, and Italy [or between some members of the EEC] are tense at
present."

The computer software being designed for a project studying native American access to higher
education not only meets the needs of that study, but also has the versatility and power of
facilitiating similar research endeavors.

a. but also has the versatility and power of facilititating


b. but also have the power to facilitate
c. but it also has the versatility and power to facilitate
d. and also have the versatility and power of facilititating
e. it also has such versatility and power that it can facilitate

The answer is
SPOILER: B
but I think it should be
SPOILER: C

EXPLANATION BY BOB:

The correctly typed sentence reads like this:

The computer software being designed for a project studying Native American access to higher education
will not only meet the needs of that study, but also have the versatility and power to facilitate similar
research endeavors.

It doesn't matter whether the subject is singular or plural. After will you must write have, never has.

35

Adhamjons SC notes

UZBEKISTAN

Q; C is still correct then, not just not idiomatically?

No. The question as typed at the top of this thread has no correct answer. "Not only meets" and "but it also
has" are not parallel. It should be "not only meets" and "but also has."

A is close, but "power of facilitating" is not idiomatic. Should be "power to facilitate."

Not only did the systematic clearing of forests in the United States create farmland (especially in the
Northeast) and gave consumers relatively inexpensive houses and furniture, but it also caused
erosion and very quickly deforested whole regions.

1. Not only did the systematic clearing of forests in the United States create farmland (especially in
the Northeast) and gave consumers relatively inexpensive houses and furniture, but it also
2. Not only did the systematic clearing of forests in the United States create farmland (especially in
the Northeast), which gave consumers relatively inexpensive houses and furniture, but also
3. The systematic clearing of forests in the United States, creating farmland (especially in the
Northeast) and giving consumers relatively inexpensive houses and furniture, but also
4. The systematic clearing of forests in the United States created farmland (especially in the
Northeast) and gave consumers relatively inexpensive houses and furniture, but it also
5. The systematic clearing of forests in the United States not only created farmland (especially in the
Northeast), giving consumers relatively inexpensive houses and furniture, but it

EXPLANATION BY BOB:

B is no good. "Not only did the systematic clearing... create..." doesn't parallel "but also gave..."
Furthermore, B erroneously uses "which" to refer to the whole preceding clause.

The correct response is D.

ANOTHER EXPLANATION BY BOB:

I think the best choice is D. The first clause says that the systematic clearing did some good things
(created farmland and provided inexpensive houses and furniture); the second clause says that it also did
36

Adhamjons SC notes

UZBEKISTAN

some bad things (caused erosion and deforested whole regions). "Not also" generally calls for a later "but
also," but "but also" does not necessarily call for an earlier "not only." Example:

Rio de Janeiro has some of the world's most famous beaches, but it also has some of the world's most
notorious slums

New hardy varieties of rice show promise of producing high yields without the costly requirements
of irrigation and application of commercial fertilizer by earlier high-yielding varieties.

(A) requirements of irrigation and application of commercial fertilizer by earlier high-yielding


varieties

(B) requirements by earlier high-yielding varieties of application of commercial fertilizer and


irrigation

(C) requirements for application of commercial fertilizer and irrigation of earlier high-yielding
varieties

(D) application of commercial fertilizer and irrigation that was required by earlier high-yielding
varieties

(E) irrigation and application of commercial fertilizer that were required by earlier high-yielding
varieties

EXPLANATION BYBOB:

The word order in C is really bad. "Costly" is placed before and therefore modifies "requirements" instead
of "irrigation and application of commercial fertilizer." Does "application of commercial fertilizer and
irrigation" mean "(1) application of commercial fertilizer and (2) irrigation" or does it mean "application
of (1) commercial fertilizer and (2) irrigation"? And then there's the placement of the phrase "of earlier
high-yielding varieties." Does only "irrigation" relate to the earlier varieties, or do both "application" and
"irrigation" relate to them?

37

Adhamjons SC notes

UZBEKISTAN

What's wrong with E? Passive? So what? It's a persistent myth that passive is bad. Sometimes passive is
not only acceptable, but necessary. Look for example at OG11 SC questions 67 and 86

By the time peace and happiness will have come to the planet,many lives will be wasted.

A......................
B. come to the planet, many lives will have been wasted
C. will have come to the planet, many lives will have been wasted
D. shall have come to the planet, many lives shall be wasted
E. would have come to the planet, many lives would have been wasted

EXPLANATION BY BOB:

I agree with the OA.

In the first clause we need the present tense. With "by the time," as with "when," "before," "while,"
"after," and "until," we use the present with a future meeting.

By the time we get there, the store will be closed.


When I see you tomorrow, I will give you the dollar I owe you.
He will turn off the lights before he leaves.
Will you take care of the plants while I am on vacation?
We won't go until we are finished.

So already we see that the correct response must be B.

In the second clause the future perfect "will have been" is correct. The future perfect refers to an event that
happens in the past from the point of view of the future. The time when peace and happiness come is in
the future. Lives will be wasted before that time.

38

Adhamjons SC notes

UZBEKISTAN

One pervasive theory explains the introduction of breakfast cereals in the early 1900s as a result of
the growing number of automobiles, which led to a decline in horse ownership and a subsequent
grain glut; by persuading people to eat what had previously been horse feed, market equilibrium
was restored.

(A) by persuading people to eat what had previously been horse feed, market equilibrium was
restored

(B) persuading people to eat what had previously been horse feed restored market equilibrium

(C) by persuading people to eat what had previously been horse feed, it restored market equilibrium

(D) the persuasion of people to eat what had previously been horse feed restored market
equilibrium

(E) market equilibrium was restored when people were persuaded to eat former horse feed

EXPLANATION BY BOB:

The correct response is indeed B.

A is wrong because the phrase "by persuading..." illogically modifies "market equilibrium."

C is wrong because "it" has no logical antecedent.

D is wrong because "persuasion of people" is ambiguous. Are the people persuading or are they being
persuaded?

E is wrong for a couple of reasons:


1) The word "when" suggests that "market equilibrium was restored" and "people were persuaded" merely
happened concurrently. The writer's intent is to say that "people were persuaded" was the cause of "market
equilibrium was restored."
2) "Former horse feed" is awkward and imprecise. Sounds like feed for former horses.
39

Adhamjons SC notes

UZBEKISTAN

As the honeybee's stinger is heavily barbed, staying where it is inserted, this results in the act of
stinging causing the bee to sustain a fatal injury.

A.As the honeybee's stinger is heavily barbed, staying where it is inserted, this results in the act of
stinging causing

B.As the heavily barbed singer of the honey bee stays where it is inserted, with the result that the act
of stinging causes

C.The honeybee's stinger, heavily barbed and staying where it is inserted, results in the fact that the
act of stinging causes

D.The heavily barbed stinger of the honeybee stays where it is inserted, and results in the act of
stinging causing

E.The honeybee's stinger is heavily barbed and stays where it is inserted, with the result that the act
of stinging causes

EXPLANATION BY BOB:

I do not agree with (C). It is not the bee's stinger that causes the bee's death. It is the fact that the stinger is
barbed and stays where it is inserted. For me, the best answer is (E).

Seems like a "GMAT quality" question to me. It has four absolutely wrong answer choices and one
answer choice with absolutely nothing wrong.

(A) is wrong because it uses "this" to refer not to a specific noun but to the whole idea expressed earlier in
the sentence. (A) is also wrong because of "results in the act of stinging causing..." "Causing" is a gerund
and needs to be preceded by the possessive, which would be the very ugly "the act of stinging's causing..."
The only way around that would be to write something like "results in the fact that the act of stinging
causes..."

40

Adhamjons SC notes

UZBEKISTAN

(B) is wrong because it is a sentence fragment: it has no independent clause.

(C) is wrong because it illogically states that the stinger results in the fact that the act results in the bee's
death.

(D) is wrong because, like (C), it says that the stinger results in the bee's death, and because, like (A), it
fails to use the possessive before the gerund.

What's wrong with (E)?

It" does not refer to the possessive noun "honeybee's." It refers to "stinger." Nothing wrong with that. The
correct response is E.

Side note: GMAT writers do not subscribe to the rule that a pronoun cannot refer to a possessive noun.
There's a question in GMATPrep in which the correct answer has a pronoun that does just that.

A substance derived from the Madagascar periwinkle, which has proved useful in decreasing
mortality among young leukemia patients, is cultivated in China as part of a program to integrate
traditional herbal medicine into a contemporary system of health care.

(A) A substance derived from the Madagascar periwinkle, which has proved useful in decreasing
mortality among young leukemia patients,

(B) A derivative, which has proved useful in decreasing mortality among young leukemia patients,
of the Madagascar periwinkle,

(C) A Madagascar periwinkle derivative, which has proved useful in decreasing mortality among
young leukemia patients,

(D) The Madagascar periwinkle has a derivative which has proved useful in decreasing mortality
among young leukemia patients, that

41

Adhamjons SC notes

UZBEKISTAN

(E) The Madagascar periwinkle, a derivative of which has proved useful in decreasing mortality
among young leukemia patients,

EXPLANATION BY BOB:

I picked E.

I eliminated C for two reasons. First, it's the periwinkle, not the derivative, that is cultivated in China.
Second, C awkwardly and imprecisely strings three nouns together: "Madagascar periwinkle derivative."
It sounds bad to this cultivated native speaker's ear, and it's not clear what the first noun is modifying. Is is
a derivative of the Madagascar periwinkle, or is it a Madagascar derivative of the periwinkle?

1. While depressed property values can hurt some large investors, they are potentially devastating
for home-owners, whose equityin many cases representing a lifes savingscan plunge or even
disappear.
(A) they are potentially devastating for homeowners, whose
(B) they can potentially devastate homeowners in that their
(C) for homeowners they are potentially devastating, because their
(D) for homeowners, it is potentially devastating in that theirA
(E) it can potentially devastate homeowners, whose

Why is (A) correct? the use of 'they' can either represent 'proporty values' or 'large investors'. Please let me
know.

EXPLANATION BY BOB:

Plural pronouns do not necessarily refer to the nearest plural noun, and singular pronouns do not
necessarily refer to the nearest singular noun. If that were true, there would be no such thing as ambiguous
pronoun reference. In the OG alone there are dozens of SC items in which the correct answer choice
includes a pronoun whose antecedent is not the nearest available noun. In addition to the item quoted
above (SC 159), here are just six more:

42

Adhamjons SC notes

UZBEKISTAN

97. Iguanas have been an important food source in Latin America since prehistoric times, and they are
still prized as game animals by the campesinos....
The pronoun they refers to iguanas, not to the nearest plural noun times.

103. Students in the metropolitan school district are so lacking in math skills that it will be difficult to
absorb them into a city economy....
The pronoun them refers to students, not to the nearest plural noun skills.

152. ...a microbe never before seen on Earth that might escape from the laboratory and kill vast numbers
of humans who would have no natural defenses against it.
The pronoun it refers to microbe, not to the nearest singular noun laboratory.

153. A recording system was so secretly installed and operated in the Kennedy Oval Office that even
Theodore C. Sorensen, the White House counsel, did not know it existed.
The pronoun it refers to system, not to the nearest singular noun Office.

180. Quasars are so distant that their light has taken billions of years to reach the Earth; consequently,
we see them as they were during the formation of the universe.
The pronouns them and they refer to quasars, not to the nearest plural noun years.

251. The gyrfalcon, an Arctic bird of prey, has survived a close brush with extinction; its numbers are now
five times greater than when....
The pronoun its refers to gyrfalcon, not to the nearest singular noun extinction.

The rule is not so simply stated as "a pronoun will refer to the nearest available noun." In fact, as all the
above examples show, if a pronoun agrees in number with the subject of the preceding clause, then that
subject will normally be the antecedent, even if there is another noun of the same number closer to the
pronoun.

"Jose's room is so messy that HIS mother calls HIM a pig."

Per Manhattan SC, in the above sentence "HIM" is used incorrectly because it may not refer back to Jose.

43

Adhamjons SC notes

UZBEKISTAN

What else could "HIM" refer to?

For the above sentence to be grammatically correct on GMAT the fix is -

"Jose's room is so messy that HIS mother calls JOSE a pig."

Why is the objective pronoun incorrect [per GMAT terms]?

doesnt HIM indicate Jose to be the object?

Thnx.

Oh well! Never mind.

The subject here is Jose's room and not Jose, so we need to replace Him with Jose.

EXPLANATION BY BOB:

Some people believe it is inappropriate for a pronoun to refer to a noun in the possessive form (that is,
with an apostrophe s). I am certain, however, that GMAT writers do not subscribe to this rule -- I clearly
recall seeing a Sentence Correction item in GMATPrep in which the correct response has a pronoun that
refers to a possessive noun

Q: 800Bob, do you recall the other answer choices. Was there any option where the possessive noun was
not referred by a pronoun?

Yes. As I recall, the pronoun was in the non-underlined portion of the sentence and the antecedent was in
the underlined portion. Two or three answer choices had the antecedent without apostrophe+s, and the
other choices had the antecedent with apostrophe+s. The credited respose was one of the choices with the
possessive form. I remember it well, because it was the only question I answered incorrectly. Blind
allegiance to the "rule" that a pronoun may not refer to a possessive noun led me to choose an answer that
I should have known was unidiomatic.

44

Adhamjons SC notes

UZBEKISTAN

I wish I had written the question down. Next time I come across it I will do so.

Here are three correct answers from 1000SC that violate this "rule":

316. Frances Wrights book on America contrasted the republicanism of the United States with what she
saw as the aristocratic and corrupt institutions of England. [Pronoun "she" refers to "Frances Wright's"]

458. Joplins faith in his opera Tremonisha was unshakable; in 1911 he published the score at his own
expense and decided to stage the work himself. [Pronoun "he" refers to "Joplin's"]

547. On stage, the force of Carricks personality and the vividness of his acting disguised the fact that he
was, as his surviving velvet suit shows, a short man. [Pronoun "he" refers to "Carrick's"]

1. In large doses, analgesics that work in the brain as antagonists to certain chemicals have caused
psychological disturbances in patients, which may limit their potential to relieve severe pain.
(A) which may limit their potential to relieve
(B) which may limit their potential for relieving
(C) which may limit such analgesics potential to relieve
(D) an effect that may limit their potential to relieve
(E) an effect that may limit the potential of such analgesics for relieving

I have a question on this problem, I was able to get till D & E and then picked D thinking that 'their' refers
to analgesics because as told by Bob, pronoun refers to subject of previous clause, which is "analgesics" .
Also E looks to me lengthy
plz advise where i'm going wrong...thanx!!

EXPLANATION BY BOB:

I say E.

This is an interesting item. I'm not sure it's a real GMAT item, but I still think it's a useful one for
illustrating the dangers of following a rule of grammar too literally. Those of us with extremely analytical
minds would like to think that the rules of grammar can be expressed with precision. We must face the
45

Adhamjons SC notes

UZBEKISTAN

fact, however, that language is not strictly logical.

I stand by the guideline that a pronoun will tend to refer to the subject of the preceding phrase if the
pronoun and subject agree in number. (We have seen many, many examples where the pronoun does not
automatically refer to the nearest noun.) So in this case it would seem that "their" in choice D refers
grammatically and logically to "analgesics." The problem with D, however, is that there are just too many
other plural nouns between "analgesics" and "their": "antagonists," "chemicals," "disturbances,"
"patients." For this reason it is better to write "of such analgesics" than "their."

Here's what I tell my students: If you have a choice that uses a pronoun and a choice repeats the noun,
generally you should go for the one that repeats the noun. A little repetition for the sake of clarity is not a
bad thing

Lincoln, discovering in young manhood the secret that the Yankee peddler has learned before him,
knew how to use a good story to generate good will.
(A) Lincoln, discovering in young manhood the secret that the Yankee peddler has learned before
him, knew
(B) Discovering in young manhood the secret that the Yankee peddler has learned before him,
Lincoln knew
(C) Lincoln, discovering the secret that the Yankee peddler had learned in young manhood before
him, knew
(D) In young manhood Lincoln discovered the secret that the Yankee peddler had learned before
him;
(E) Lincoln, discovered in young manhood the secret that the Yankee peddler had learned before
him, knew

EXPLANATION BY BOB:

(A) Lincoln, discovering in young manhood the secret that the Yankee peddler has learned before him,
knew
(B) Discovering in young manhood the secret that the Yankee peddler has learned before him, Lincoln
knew
46

Adhamjons SC notes

UZBEKISTAN

Both wrong. Need past perfect "had learned."

(C) Lincoln, discovering the secret that the Yankee peddler had learned in young manhood before him,
knew
Wrong. "In young manhood" is misplaced.

(D) In young manhood Lincoln discovered the secret that the Yankee peddler had learned before him;
Wrong. Semicolon makes no sense here. A semicolon separates equivalent elements. Here there's an
independent clause before but a mere phrase after. If it were a colon, this sentence would be OK.

(E) Lincoln, discovered in young manhood the secret that the Yankee peddler had learned before him,
knew
Wrong. The phrase between commas begins with the past participle "discovered," seeming to
suggest at first that Lincoln was discovered, but then making no sense at all when "the secret"
appears as the object of "discovered."

So every choice is wrong. I believe that the answer is supposed to be D, but that somewhere along the way
into 1000SC a colon got changed into a semicolon.

Some psychiatric studies indicate that among distinguished artists the rates of manic depression and
major depression are ten to thirteen times as prevalent as in the population at large.
(A) the rates of manic depression and major depression are ten to thirteen times as prevalent as in
(B) the rates of manic depression and major depression are ten to thirteen times more
prevalent than in
(C) the rates of manic depression and major depression are ten to thirteen times more
prevalent when compared to
(D) manic depression and major depression are ten to thirteen times as prevalent when compared to
(E) manic depression and major depression are ten to thirteen times more prevalent than in

EXPLANATION BY BOB:

47

Adhamjons SC notes

UZBEKISTAN

"rates...are more prevalent" is redundant and imprecise. Say either "the rates of manic depression and
major depression are greater" or "manic depression and major depression are more prevalent." It doesn't
really make sense to say that "rates" are "more prevalent."
Rate can never be fast/slow/prevalent...it is always greater or lesser...

The proposed health care bill would increase government regulation of health insurance, establish
standards that would gurantee wider access to people with past health problems and to workers
changing jobs who otherwise could be uncovered for months.

A) establish standards that would gurantee wider access to people with past health problems and to
workers changing jobs who

B) establishing standards that would gurantee wider access to people with past health problems and
to workers who are changing jobs and

C) to establish standards that would gurantee wider access to people with past health problems and
to workers who change jobs that

D) for establishing standards that would gurantee wider access for people with past health problems
and workers changing jobs who

E) for the establishment of standards that would gurantee wider access for people with past health
problems and workers who are changing jobs that

OA-B

EXPLANATION BY BOB:

In Question 8, A is impossible:

The proposed health care bill would increase government regulation of health insurance, establish
standards that would gurantee wider access to people with past health problems and to workers changing
48

Adhamjons SC notes

UZBEKISTAN

jobs who
otherwise could be uncovered for months.

In this would-be sentence, we have "the... bill would increase..., establish..." You need "and" before
"establish." You can't say "the bill would increase regulations, establish standards..." (unless there's
another verb later -- but there's not). It should be "the bill would increase regulations and establish
standards..."

What's wrong with B? "...wider access (1) to people with past health problems and (2) to workers who (a)
are changing jobs and (b) otherwise could be uncovered for months."

It is an oversimplified view of cattle raising to say that all one has to do with cattle is leave them
alone while they feed themselves, corral them and to drive them to market when the time is ripe.

A) all one has to do with cattle is leave them alone while they feed themselves, coral them and to

B) all one has to do with cattle is to leave them alone to feed themselves, to corral them, and

C) all one has to do with cattle is leave them alone while they feed themselves and then corral them
and

D) the only thing that has to be done with cattle is leave them alone while they feed themselves,
corral them, and

E) the only thing that has to be done with cattle is to leave them alone while ttey feed themselves, to
corral them, and

EXPLANATION BY BOB:

In Question 11, A, B, and E are easily eliminated for lack of parallelism. They all attempt to present a list
of three verbs. To do so, you should repeat "to" always or never. That is, either:

"to leave..., to corral..., and to drive..."


49

Adhamjons SC notes

UZBEKISTAN

or:

"to leave..., corral..., and drive..."

That leaves C and D.

D has a couple of problems. It says "the only thing that has to be done..." Then it lists three verbs. "Only
thing" implies one, not three. And verbs are not things. You can use an infinitive or a gerund as a noun,
but not the base form of the verb.

Oberlin College in Ohio was a renegade institution at its 1833 founding for deciding to accept both
men and women as students

A) at its 1833 founding for deciding to accept

B) for the decision at its 1833 founding to accept

C) when it was founded in 1833 for its decision to accept

D) in deciding at its founding in 1833 to accept

E) by deciding at its fouding in 1833 on the acceptance of

OA-D

Last year, land values in most parts of the pinelands rose almost so fast, and in some parts even
faster than what they did outside the pinelands.

A) so fast, and in some parts even faster than what they did

B) so fast, and in some parts even faster than, those


50

Adhamjons SC notes

UZBEKISTAN

C) as fast, and in some parts even faster than, those

D) as fast as, and in some parts even faster than, those

E) as fast as, and in some parts even faster than what they did

EXPLANATION BY BOB:

D "What they did" is imprecise and unnecessary. You could say, for example, that "John ran as fast as, or
even faster than, Paul" or "John ran as fast as, or even faster than, Paul ran" or "John ran as fast as, or even
faster than, Paul did." But you can't write "John ran as fast as, or even faster than, what Paul did."

In the mid-1960s a newly installed radar warning system mistook the rising of the moon as a
massive missile attack by the Soviets.

A) rising of the moon as a massive missile attack by the Soviets

B) rising of the moon for a massive Soviet missile attack

C) moon rising to a massive missile attack by the soviets

D) moon as it was rising for a massive Soviet missile attack

E) rise of the moon as a massive Soviet missile attack

EXPLANATION BY BOB:

B The correct idiom is "mistake x for y," as in the book title "The Man Who Mistook His Wife for a Hat."

51

Adhamjons SC notes

UZBEKISTAN

With only 5 percent of the worlds population, United States citizens consume 28 percent of its
nonrenewable resources, drive more than one-third of its automobiles, and use 21 times more water
per capita than Europeans do.
A) With
B) As
C) Being
D) Despite having
E) Although accounting for

EXPLANATION BY BOB:

E The subject is "United States citizens." You can't write "with" or "having" because citizens don't have a
population. You can't write "as" or "being" because they are not a number. The only choice that makes
sense is E: United States citizens "account for" 5 percent of the world's population.

Neanderthals had a vocal tract that resembled those of the apes and so were probably without
language, a shortcoming that may explain why they were supplanted by our own species.

A) Neanderthals had a vocal tract that resembled those of the apes

B) Neanderthals had a vocal tract resembling an apes

C) The vocal tracts of the Neanderthals resembled an apes

D) The Neanderthals vocal tracts resembled the apes

E) The vocal tracts of the Neanderthals resembled those of the apes

EXPLANATION BY BOB:

B Singular "vocal tract" can be compared to "that of the apes" or "that of an ape" or "an ape's," but not to
plural "those of the apes."
52

Adhamjons SC notes

UZBEKISTAN

Q: B states that "Vocal Tract resembles an ape's ", so here vocal tract resembles to what part of ape's ? Is
this clear in this question . I dont know what i am asking might be very basic

"...had a vocal tract resembling an ape's..." means "...had a vocal tract resembling an ape's vocal tract."
You can use the 's possessive to stand for the noun that's possessed, just as you can use possessive
pronouns such as "mine," "yours," and "hers."

The baby has a face resembling his mother's face.


The baby has a face resembling his mother's.

The baby has a face resembling your face.


The baby has a face resembling yours.

Heavy commitment by an executive to a course of action, especially if it has worked well in the past,
makes it likely to miss signs of incipient trouble or misinterpret them when they do appear.

(A ) Heavy commitment by an executive to a course of action, especially if it has worked well in the
past, makes it likely to miss signs of incipient trouble or misinterpret them when they do appear.

(B ) An executive who is heavily committed to a course of action, especially one that worked well in
the past, makes missing signs of incipient trouble or misinterpreting ones likely when they do
appear.

(C ) An executive who is heavily committed to a course of action is likely to miss or misinterpret


signs of incipient trouble when they do appear, especially if it has worked well in the past.

(D ) Executives being heavily committed to a course of action, especially if it has worked well in the
past, makes them likely to miss signs of incipient trouble or misinterpreting them when they do
appear.

(E ) Being heavily committed to a course of action, especially one that has worked well in the past, is

53

Adhamjons SC notes

UZBEKISTAN

likely to make an executive miss signs of incipient trouble or misinterpret them when they do
appear.

BOB:

Your "OA" is wrong. The correct response is E.

I am confused between A & E.


please somebody explain the usage of gerund & infinitive.
what difference it makes in this particular case.

According to some analysts, whatever its merits, the proposal to tax away all capital gains on shortterm investments would, if enacted, have a disastrous effect on Wall Street trading and employment.
(A) its merits, the proposal to tax
(B) its merits may be, the proposal of taxing
(C) its merits as a proposal, taxing
(D) the proposals merits, to tax
(E) the proposals merits are, taxing

EXPLANATION BY BOB:

The deciding difference between A and E is not infinitive versus gerund. What makes E wrong is that
"taxing" makes no sense as the subject. Later in the sentence we see "if enacted," referring to the subject.
It makes no sense to speak of what would happen if taxing is enacted. The sentence is speculating on what
would happen if the proposal is enacted

One of Ronald Reagans first acts as President was to rescind President Carters directive that any
chemical banned on medical grounds in the United States be prohibited from sale to other countries.
(A) that any chemical banned on medical grounds in the United States be prohibited from sale to
other countries
(B) that any chemical be prohibited from sale to other countries that was banned on medical
54

Adhamjons SC notes

UZBEKISTAN

grounds in the United States


(C) prohibiting the sale to other countries of any chemical banned on medical grounds in the United
States
(D) prohibiting that any chemical banned on medical grounds in the United States is sold to other
countries
(E) that any chemical banned in the United States on medical grounds is prohibited from being sold
to other countries.

I know its already dicussed but my question is that isn't the sentence subjuntive by the word - 'directive'?
Should it not be followed by 'that...'. Please Help me clarify on this

EXPLANATION BY BOB:

When a verb, noun, or adjective of recommendation, requirement, or request is followed by a "that"


clause, the verb in the "that" clause will be in the subjunctive mood. But a verb, noun, or adjective of
recommendation, requirement, or request does not have to be followed by a "that" clause.

In the item under discussion, choice A nonsensically states that the directive prohibits chemicals from
doing something. You can't tell chemicals what to do or not to do. You can prohibit the sale of chemicals,
but you can't prohibit chemicals from being sold

A recent New York Times editorial criticized the citys elections board for, first of all, failing to
replace outmoded voting machines prone to breakdowns, and secondarily, for their failure to
investigate allegations of corruption involving board members.
1) secondarily, for their failure to
2) secondly, for their failure to
3) secondly, that they failed and did not
4) second, that they failed to
5) second, for failing to

55

Adhamjons SC notes

UZBEKISTAN

EXPLANATION BY BOB:

The correct response is E. All the other choices are easily eliminated because "their" and "they" do not
agree with the singular antecedent "board."

Violence in the stands at soccer matches has got so pronounced in several European countries that
some stadiums have adopted new rules that aim to identify fans of visiting teams and that seat them
in a separate area.
1. ----------------------------------------2. to identify fans of visiting teams and seat them
3. to identify fans of visiting teams for seating
4. at identifying fans of visiting teams so as to seat them
5. at identifying fans of visiting teams and that seat them

OA-B

a) to identify fans and that seat ...( that is a pronoun following and .... wrong ....infinitive to pronoun
comparision .
b) to identify and seat ( to is implicit , infinitive to infinitive comparision ) correct
c) and is missing , also not parallel (to identify .... for seating )
d) wordy unidiomatic and not parallel (...identifying ....to seat ...)
e) not parallel (identifying and that seat )

Hence B is correct

Hi

A majority of the voters want/wants to unseat the incumbent.

plz xplain

56

Adhamjons SC notes

UZBEKISTAN

EXPLANATION BY BOB:

A majority of the voters is plural, as are:

one half of the voters


more than one half of the voters
a lot of the voters
a number of the voters

Here are some examples taken from the OG, 10th edition.

CR 23 (C) The majority of travelers departing from rural airports are flying to destinations in cities over
600 miles away.

CR 44 (D) the majority of the cases of sex discrimination against women that have reached the judge's
court have been appealed from a lower court

CR 136 (B) The majority of the magazine's subscribers are less concerned about a possible reduction
in the quantity of the magazine's articles than about a possible loss of the current high quality of its
articles.

CR 148 (E) The majority of the retailer's employees work as cashiers, and most cashiers are paid the
minimum wage.

CR 194 (B) Because a majority of commuters drive at least 100 miles a week, the cost to the producer
of making good the offer would add considerably to the already high price of electric vehicles.

SC 13 A majority of the international journalists surveyed view nuclear power stations as unsafe at
present...

FROM OTHER GUY:

I think correct sentence should be "A majority of the voters want to unseat the incumbent"

57

Adhamjons SC notes

UZBEKISTAN

A little hint: The before the word (the total, the majority) is usually a tip-off that it's singular, while a (a
total, a number, a majority), especially when of comes after, usually indicates a plural.

It is well known in the supermarket industry that how items are placed on shelves and the
frequency of inventory turnovers can be crucial to profits.

(A) the frequency of inventory turnovers can be


(B) the frequency of inventory turnovers is often
(C) the frequency with which the inventory turns over is often
(D) how frequently is the inventory turned over are often
(E) how frequently the inventory turns over can be

EXPLANATION BY BOB:

Quote:
Originally Posted by amitg_ind
Whats wrong with D?
When a question word ("what," "where," "when," "how," etc.) is used in a statement (as opposed to a
question) it is incorrect to invert subject and verb.

Examples:
What time is it?
I don't know what time it is. (It would be wrong to say "I don't know what time is it.")

Where did he go?


He did not tell us where he went. (It would be wrong to say "He did not tell us where did he go.")

When will he arrive?


I don't know when he will arrive. (It would be wrong to say "I don't know when will he arrive.")

How frequently is the inventory turned over?


How frequently the inventory is turned over can be crucial. (It would be wrong to say "How frequently is
the inventory turned over can be crucial.)
58

Adhamjons SC notes

UZBEKISTAN

Is this scentence formation right?

"JUST as General Motors has long been an icon of American business, so Volkswagen is a symbol
of the German industrial economy."

I got the above scentence from an article on the Economist.com website.

EXPLANATION BY BOB:

The structure is perfectly idiomatic. Here's a similar sentence from OG11:

Just as melting ice cubes do not cause a glass of water to overflow, so melting sea ice does not increase
oceanic volume.

Antarctica receives more solar radiation than does any other place on Earth, yet the temperatures
are so cold and the ice cap is reflective, so that little polar ice melts during the summer; otherwise,
the water levels of the oceans would rise 250 feet and engulf most of the world's great cities.
A. is reflective, so that little polar ice melts during the summer; otherwise,
B. is so reflective that little of the polar ice melts during the summer; were it not to do so,
C. so reflective that little polar ice melts during the summer, or else
D. reflective, so that little of the polar ice melts during the summer, or
E. reflects so that little of the polar ice melts during the summer; if it did

EXPLANATION BY BOB:

B is incorrectly typed. It should read:

is so reflective that little of the polar ice melts during the summer; were it to do so,

With "not" removed, B is correct. The problem with C is "or else." Meaning "if not" or "otherwise," the
59

Adhamjons SC notes

UZBEKISTAN

words "or else" don't make sense here. With C, the last part of the sentence is saying that, if that little bit
of polar ice did not melt in the summer, the ocean levels would rise. What the sentence is trying to convey
is that if more than a little polar ice melted the ocean levels would rise.

For many travelers, charter vacations often turn out to cost considerably more than they originally
seemed.
(A) they originally seemed
(B) they originally seem to
(C) they seemingly would cost originally
(D) it seemed originally
(E) it originally seemed they would

EXPLANATION BY BOB:

The OA is wrong. The correct response is E. "Charter vacations often turn out to cost considerably more
than it originally seemed they would [cost]."

"It" here has no antecedent. It's not really a pronoun. It's subject of the impersonal verb "seemed."

It originally seemed that the charter vacations would not cost very much.

Here are some other examples of the impersonal use of "it":

It is nine o'clock.
It was raining.
It feels cold here.

Quote:
Originally Posted by chatru
800Bob: how do you justify using "it" in your asnwer."what antecedent is refered by "It" in your choice of
answer "E".

60

Adhamjons SC notes

UZBEKISTAN

No antecedent. Read what I wrote above. What's the antecedent of "it" in the following?

I looked out the window and saw that it was raining.


Quote:
Originally Posted by drctypea
thx bob

but why do we even need the word "it." What is wrong with an answer choice like B?

Recently documented examples of neurogenesis, the production of new brain cells, include the brain
growing in mice when placed in a stimulating environment or neurons increasing in canaries that
learn new songs.

A. the brain growing in mice when placed in a stimulating environment or neurons increasing in
canaries that

B. mice whose beains grow when they are placed in stimulating environment or canaries whose
neurons increase when they

C. mice's brains that grow when they are placed in stimulating environment or canaries' neurons
that ncrease when they

D. the brain growth in mice when placed in a stimulating environment or the increase in canaries'
neuons when they

E. brain growth in mice that are placed in a stimulating environment or an increase in neurons in
canaries that

EXPLANATION BY BOB:

61

Adhamjons SC notes

UZBEKISTAN

"Recently documented examples of neurogenesis... include..." must be followed by two nouns or noun
phrases that are examples of neurogenesis. That narrows the choices to D and E.

D has a couple of problems. First, it is not clear what is "placed in a stimulating environment." Second, the
reference of "they" is ambiguous. Does it refer to "neurons"? That doesn't make sense. Does it refer to
"canaries"? It's not good to have a subject pronoun refer to a noun that's in the possessive case.

The best choice is E.

Although he is as gifted as, if not more gifted than, many of his colleagues, he is extremely modest
and his poetry is unpublished.

(A) Although he is as gifted as, if not more gifted than, many of his colleagues, he is extremely
modest and his poetry is unpublished.

(B) Although he is as gifted, if not more gifted, than many of his colleagues, he is extremely modest
and with his poetry remaining unpublished.

(C) Although he is as gifted as, if not more gifted than, many of his colleagues, he is extremely
modest and will not publish his poetry.

(D) Despite his being gifted, if not more gifted than his colleagues, he is extremely modest and will
not publish his poetry.

(E) Being a gifted as, or more gifted than, many of his colleagues, he is extremely modest and his
poetry is unpublished

DISPUTE IS OVER A AND C

EXPLANATION BY BOB:

Don't obsess so much about whether an answer choice "changes the meaning." Sometimes it is necessary
to change the meaning of the original, such as when the original is ambiguous, illogical, or confused. For
62

Adhamjons SC notes

UZBEKISTAN

me the first sentence is trying to say that his poetry is unpublished because he is modest. Choice C makes
more sense.

The point of the Sentence Correction game is to avoid changing the intended meaning. The challenge is to
determine from a poorly written sentence what exactly the author intends

Although the manager agreed to a more flexible work schedule, he said that it must be posted on the
bulletin board so that both management and labor will know what everyone is assigned to do.

(A) he said that it must be posted on the bulletin board so that both management and labor will
know what everyone is

(B) he said it had to be posted on the bulletin board so that both management and labor knows what
everyone is

(C) he said that they would have to post the assignments on the bulletin board so that management
and labor knew what everyone was

(D) he said that the schedule would have to be posted on the bulletin board so that both
management and labor would know what everyone was

(E) saying that the schedule had to be posted on the bulletin board so that both management and
labor would know what everyone had been

OA-A

Coronary angiography, a sophisticated method for diagnosing coronary disease involving the
introduction of a dye into the arteries of the heart, is now administered selectively, because it uses xrays to observe cardiac function.
(A) for diagnosing coronary disease involving the introduction of a dye into the arteries of the heart,
is now administered selectively, because it uses
(B) for diagnosing coronary disease involving the introduction of a dye into the arteries of heart, is
63

Adhamjons SC notes

UZBEKISTAN

now administered selectively, because of using


(C) for diagnosing coronary disease, involves the introduction of dye into the arteries of the heart
and is now administered selectively, because it uses
(D) to diagnose coronary disease that involves the introduction of a dye into the arteries of the heart,
is now administered selectively, because it uses
(E) to diagnose coronary disease involving the introduction of a dye into the arteries of the heart,
which is now administered selectively, uses

EXPLANATION BY BOB:

The best response is C. The other choices are wrong because "coronary disease involving" and "coronary
disease that involves" both suggest that the disease involves the introduction of a dye.

Quote:
Originally Posted by dimbulb
800Bob , can u pls explain why D is wrong ?

Is the usage of "that" not required?

D is wrong because "a sophisticated method to diagnose coronary disease that involves the introduction of
a dye into the arteries of the heart" is confusing: it seems to say that coronoary disease involves the
introduction of a dye.

I don't see why "that" is necessary. In correct choice C "involves the introduction of a dye" becomes part
of the main clause. The grammatical core of the sentence is a subject and two verbs: "...angiography ...
involves ... and is..."

Legend has it that when the Mohawk Joseph Brant was presented to George III in 1776, he proudly
refused to kiss the Kings hand, inasmuch as he regarded himself an ally, not a subject.
(A) as he regarded himself an ally, not
(B) that he regarded himself to be an ally rather than
(C) as he considered himself an ally, not
(D) that he considered himself to be ally instead of
(E) as he considered himself as an ally rather than
64

Adhamjons SC notes

UZBEKISTAN

EXPLANATION BY BOB:

"Regard" and "consider" are very similar in meaning, but "regard" must be followed by "as" while
"consider" should not be followed by "as."

I regard him as an ally.


I consider him an ally.

The computer software being designed for a project studying Native American access to higher
education will not only meet the needs of that study, but also has the versatility and power of
facilitating similar research endeavors.
(A) but also has the versatility and power of facilitating
(B) but also have the versatility and power to facilitate
(C) but it also has the versatility and power to facilitate
(D) and also have the versatility and power of facilitating
(E) and it also has such versatility and power that it can facilitate

Correct response is B. Parallelism: "will not only meet... but also have."
There is no difference between singular and plural after "will." You can write "not only meets... but also
has" or "will not only meet... but also have" or "not only will meet... but also will have," but you can't
write "will not only meet... but also has."

Birds' feathers appear to have evolved from dinosaur scales well over 100 million years ago, Chinese
scientists have shown, based on a fossil bird found in the Yixian mountains of northern China.
A) Chinese scientists have shown, based on a fossil bird found in the Yixian mountains of northern
China.
B) Chinese scientists have shown a fossil bird found in the Yixian mountains of northern China.
C) Chinese scientists showed, based on a fossil bird found in the Yixian mountains of northern
China.
D) when the Chinese scientists found a fossil bird in the Yixian mountains of northern China.

65

Adhamjons SC notes

UZBEKISTAN

E) which the Chinese scientists say can be proven from a fossil bird which they found in the Yixian
mountains of northern China.

EXPLANATION BY BOB:

This question is not of GMAT quality. The so-called OA is impossible. As gschmilinsky asks, what does
the phrase "based on..." modify? "Scientists"? That doesn't make any sense. You can't say that scientists
are based on a fossil bird. For a similar dangling modifier, look at this SC item from the OG:
Based on accounts of various ancient writers, scholars have painted a sketchy picture...
The correct response changes the modifying phrase to the more logical:
Using accounts of various ancient writers...

The first commercially successful drama to depict Black family life sympathetically and the first
play by a Black woman to be produced on Broadway, it was Lorraine Hansberrys
A Raisin in the Sun that won the New York Drama Critics Circle Award in 1959, and was later
made into both a film and a musical.
A. it was Lorraine Hansberrys A Raisin in the Sun that won the New York Drama Critics Circle
Award in 1959, and was later made
B. in 1959 A Raisin in the Sun, by Lorraine Hansberry, won the New York Drama Critics Circle
Award and was later made
C. Lorraine Hansberry won the New York Drama Critics Circle Award for A Raisin in the Sun in
1959, and it was later made
D. Lorraine Hansberrys A Raisin in the Sun won the New York Drama Critics
Circle Award in 1959 and was later made
E. A Raisin in the Sun, by Lorraine Hansberry, won the New York Drama Critics Circle Award in
1959, and later made it

EXPLANATION BY BOB:

No way E is acceptable. The OA is wrong. The correct response is D.

66

Adhamjons SC notes

UZBEKISTAN

X won...and was made...-PARALLELISM

Over 75 percent of the energy produced in France derives from nuclear power, while in
Germany it is just over 33 percent.
A. while in Germany it is just over 33 percent
B. compared to Germany, which uses just over 33 percent
C. whereas nuclear power accounts for just over 33 percent of the energy produced
in Germany
D. whereas just over 33 percent of the energy comes from nuclear power in Germany
E. compared with the energy from nuclear power in Germany, where it is just over
33 percent

CORRECT ANSWER IS C

Not one of the potential investors is expected to make an offer to buy First Interstate Bank until a
merger agreement is signed that includes a provision for penalties if the deal were not to be
concluded.

A. is expected to make an offer to buy First Interstate Bank until a merger agreement is

signed

that includes a provision for penalties if the deal were


B. is expected to make an offer for buying First Interstate Bank until they sign a merger agreement
including a provision for penalties if the deal was
C. is expected to make an offer to buy First Interstate Bank until a merger agreement be signed by
them with a provision for penalties if the deal were
D. are expected to make an offer for buying First Interstate Bank until it signs a merger agreement
with a provision for penalties included if the deal was
E. are expected to be making an offer to buy First Interstate Bank until they sign a merger
agreement including a provision for penalties if the deal were

EXPLANATION BY BOB:

67

Adhamjons SC notes

UZBEKISTAN

The OA is wrong. In contemporary English we do not use the subjunctive in a clause beginning with
"until." True, in French one would use the subjunctive after "jusqu'a' ce que," and in Italian one would use
the subjunctive after "finche'," but in English we use the subjunctive only in clauses of recommendation,
requirement, and request, and only in clauses introduced by "that."
The correct response is A.
What is the source of this item? Who came up with this "OA"? I can't imagine any native speaker of
modern English ever using "be" after "until."

Several senior officials spoke to the press on condition that they not be named in the story.
(A) that they not be named
(B) that their names will not be used
(C) that their names are not used
(D) of not being named
(E) they will not be named

EXPLANATION BY BOB:

The correct response is A. "Condition" is a noun that means "requirement" and therefore takes the
subjunctive. Examples:

Mr. Bush spoke on the record, but others talked to Woodward on condition that he not reveal their
identities.

He was given a suspended sentence on condition that he leave the country.

An article in Le Monde that said that Milosovic would agree to NATO terms on condition that he be
granted immunity.

Cooperative apartment houses have the peculiar distinction of being dwellings that must also
operate as businesses.
(A) of being dwellings that must also operate as businesses
(B) of dwellings that must also operate like business
68

Adhamjons SC notes

UZBEKISTAN

(C) that they are dwellings that must operate like business
(D) that, as dwellings, they must also operate like businesses
(E) to be a dwelling that must also operate as a business

OA is A.

EXPLANATION BY BOB:

As" is better than "like" here because it is not a comparison. Here "as" is a preposition meaning "in the
capacity of." There is a difference between "he works like a firefighter" and "he works as a firefighter." In
the first instance, he is not actually a firefighter but only similiar to one. In the second instance, he is a
firefighter. In the Sentence Correction item at issue, coops are not being compared to businesses; they are
businesses.

Following the destruction of the space shuttle Challenger, investigators concluded that many key
people employed by the National Aeronautics and Space Administration and its contractors work
an excessive amount of overtime that has the potential of causing errors in judgment.

(A) overtime that has the potential of causing

(B) overtime that has the potential to cause

(C) overtime that potentially can cause

(D) overtime, a practice that has the potential for causing

(E) overtime, a practice that can, potentially, cause

OA is E

EXPLANATION BY BOB:

69

Adhamjons SC notes

UZBEKISTAN

A, B, and C are all wrong because "that" would refer to "overtime." It is not the overtime that can
potentially cause errors in judgment; it is the fact that employees of NASA and contractors are required to
work an excessive amount of overtime. The end of the sentence needs a noun like "practice" to refer to the
real cause. Between D and E, I would choose E because "potential for causing" is unidiomatic -- it should
be "potential to cause."

A study of couples' retirement transitions found that women who took new jobs after retiring from
their primary careers reported high marital satisfaction, more than when retiring completely.

a]

b] more than if they were to retire

c] more so than those who retired

d] which was more so than those retiring

e] which was more than if they had retired

The OA is C.

EXPLANATION BY BOB:

What's wrong with "more so"?

For me the best answer is C. The sentence is comparing "women who took new jobs" with "those who
retired."
Agree with C.
Examples with: "more so than"

1) They talk about it all the time, but never more so than lately.
2) We take our business seriously, more so than any other company in the town.
3) "Lord of the Rings" was awesome, more so than any other movie I have recently watched, etc...
70

Adhamjons SC notes

UZBEKISTAN

Quote:
Originally Posted by CTG1983
Bob, can u plz elaborate the use of "more so" with some examples?

Look at GMZ's examples above. Or type "more so" in Google and find more than 19,000,000 more.
Actually, "more so" is such a common collocation that some people think it's one word. Type "moreso" in
Google and you'll find over a million instances.

"So" here is an adverb meaning "in such a manner, in that way." "More so" is the comparative of the
adverb.

Many of them chiseled from solid rock centuries ago, the mountainous regions of northern Ethiopia
are dotted with hundreds of monasteries.
(A) Many of them chiseled from solid rock centuries ago, the mountainous regions of northern
Ethiopia are dotted with hundreds of monasteries.
(B) Chiseled from solid rock centuries ago, the mountainous regions of northern Ethiopia are dotted
with many hundreds of monasteries.
(C) Hundreds of monasteries, many of them chiseled from solid rock centuries ago, are dotting the
mountainous regions of northern Ethiopia.
(D) The mountainous regions of northern Ethiopia are dotted with hundreds of monasteries, many
of which are chiseled from solid rock centuries ago.
(E) The mountainous regions of northern Ethiopia are dotted with hundreds of monasteries, many
of them chiseled from solid rock centuries ago.

EXPLANATION BY BOB:

Quote:
Originally Posted by amitg_ind
OA is E. But I am still confused about the usage of 'them'. 'many of them' is ambigous and hence cannot be
correct? Could anyone please throw some light on this?
We TM'ers tend to make too big a deal of potentially ambiguous pronoun reference. No sensible reader
would ever think for a second that "many of them chiseled from solid rock" could refer to "mountainous
71

Adhamjons SC notes

UZBEKISTAN

regions." And I don't think a native speaker would ever use the progressive form here: "are dotting the
mountainous regions."

Margaret Courtney-Clarke has traveled to remote dwellings in the Transvaal to photograph the art
of Ndebele women, whose murals are brilliantly colored, their geometrical symmetries embellished
with old and new iconography and in a style that varies from woman to woman and house to house.

(A) whose murals are brilliantly colored, their geometrical symmetries embellished with old and
new iconography and in a style that varies from woman to woman and house to house

(B) whose murals are brilliantly colored, their geometrical symmetries are embellished with old and
new iconography, and their style is varying among women and houses

(C) whose murals are brilliantly colored, their geometrical symmetries are embellished with old and
new iconography, and they are in styles that vary from woman to woman and house to house

(D) with murals brilliantly colored, their geometrical symmetries embellished with old and new
iconography, and their style varies among women and houses

(E) with murals that are brilliantly colored, their geometrical symmetries embellished with old and
new iconography, and their styles vary among women and houses

OA is A.

EXPLANATION BY BOB:

Must be A.

B and C are run-on sentences.

(B) [Margaret Courtney-Clarke has traveled to remote dwellings in the Transvaal to photograph the art of
Ndebele women, whose murals are brilliantly colored,] break [their geometrical symmetries are
embellished with old and new iconography, and their style is varying among women and houses.]
72

Adhamjons SC notes

UZBEKISTAN

(C) [Margaret Courtney-Clarke has traveled to remote dwellings in the Transvaal to photograph the art of
Ndebele women, whose murals are brilliantly colored,] break [their geometrical symmetries are
embellished with old and new iconography, and they are in styles that vary from woman to woman and
house to house.]

D and E have breakdowns in parallelism:

(D) ...their geometrical symmetries embellished with old and new iconography ... and their style varies
among women and houses

(E) ...their geometrical symmetries embellished with old and new iconography ... and their styles vary
among women and houses

In both cases we have a noun phrase ("symmetries embellished") and a clause with a subject and
conjugated verb ("style varies" and "styles vary").

Quote:
Originally Posted by Seth
Why B and C are run on? Anyone please clarify me?

I can't see that they are run on.

Two independent clauses joined with no more than a comma make a run-on sentence. With choice B you
have:

Margaret Courtney-Clarke has traveled to remote dwellings in the Transvaal to photograph the art of
Ndebele women, whose murals are brilliantly colored, their geometrical symmetries are embellished with
old and new iconography, and their style is varying among women and houses.

That's two independent clauses. Each can stand alone:

1. Margaret Courtney-Clarke has traveled to remote dwellings in the Transvaal to photograph the art of
73

Adhamjons SC notes

UZBEKISTAN

Ndebele women, whose murals are brilliantly colored.

2. Their geometrical symmetries are embellished with old and new iconography, and their style is varying
among women and houses.

With choice C you have:

Margaret Courtney-Clarke has traveled to remote dwellings in the Transvaal to photograph the art of
Ndebele women, whose murals are brilliantly colored, their geometrical symmetries are embellished with
old and new iconography, and they are in styles that vary from woman to woman and house to house.

That's two independent clauses. Each can stand alone:

1. Margaret Courtney-Clarke has traveled to remote dwellings in the Transvaal to photograph the art of
Ndebele women, whose murals are brilliantly colored.

2. Their geometrical symmetries are embellished with old and new iconography, and they are in styles
that vary from woman to woman and house to house.

Quote:
Originally Posted by gvirk1
Bob, Help me in understanding the usage of "their" in "their geometrical symmetries". "their" can refer to
both women and murals. Is it OK?
Or I am missing something...
Thnx

"Their" clearly refers to "murals" and not to "women" because "murals" is not only closer but also, more
crucially, in a stronger position (subject of the preceding clause) than "women" (object of a preposition).

The clearest indication that the reference is not ambiguous is that every answer choice has "their."

74

Adhamjons SC notes

UZBEKISTAN

The nineteenth-century chemist Humphry Davy presented the results of his early

experiments in his Essay on Heat and Light, a critique of all chemistry since Robert

Boyle as well as a vision of a new chemistry that Davy hoped to found.

A. a critique of all chemistry since Robert Boyle as well as a vision of a

B. a critique of all chemistry following Robert Boyle and also his envisioning of a

C. a critique of all chemistry after Robert Boyle and envisioning as well

D. critiquing all chemistry from Robert Boyle forward and also a vision of

E. critiquing all the chemistry done since Robert Boyle as well as his own

EXPLANATION BY BOB:

Quote:
Originally Posted by wakau
Who/What is the critique ?? Is it the experiments or Humphry Davy ??
Neither. "Essay on Heat and Light" is the critique.

Quote:
Originally Posted by sumeet_rana
since Robert Boyle as well as a vision of a new chemistry that Davy hoped to found.
I couldnt make any sense out of the above sentence ! therefore not A...

Davy's Essay on Heat and Light is two things:


1. a critique of all chemistry since Robert Boyle; and
2. a vision of a new chemistry that Davy hoped to found.
Hence: ... Essay on Heat and Light, a critique of all chemistry since Robert Boyle as well as a vision of
a new chemistry that Davy hoped to found.
75

Adhamjons SC notes

UZBEKISTAN

1. In recent years cattle breeders have increasingly used crossbreeding, in part that their steers
should acquire certain characteristics and partly because crossbreeding is said to provide
hybrid vigor.
(A) in part that their steers should acquire certain characteristics
(B) in part for the acquisition of certain characteristics in their steers
(C) partly because of their steers acquiring certain characteristics
(D) partly because certain characteristics should be acquired by their steers
(E) partly to acquire certain characteristics in their steers

2. In good years, the patchwork of green fields that surround the San Joaquin Valley town
bustles with farm workers, many of them in the area just for the season.
(A) surround the San Joaquin Valley town bustles with farm workers, many of them
(B) surrounds the San Joaquin Valley town bustles with farm workers, many of whom are
(C) surround the San Joaquin Valley town bustles with farm workers, many of who are
(D) surround the San Joaquin Valley town bustle with farm workers, many of which
(E) surrounds the San Joaquin Valley town bustles with farm workers, many are

EXPLANATION BY BOB:

The second question has been discussed many times, with no consensus. I personally doubt that it is a real
GMAT question because choices A and B can both be justified.

First issue:
"patchwork of green fields that surround" vs. "patchwork of green fields that surrounds"
Both are possible. They have very slightly different meanings, but both meanings make sense. In the first
case "that" is plural and refers to "fields." In the second case "that" is singular and refers to "patchwork."
In the first case the sentence says that the patchwork is made up of green fields and that the green fields
surround the town. In the second case it is the patchwork that surrounds the town. What's the difference?
Both make sense to me.

Second issue:
"many of them" vs. "many of whom are"
76

Adhamjons SC notes

UZBEKISTAN

Here is where I have a preference. "Many of whom are" is perfectly grammatical and unambiguous. In
"many of them," however, one could argue that "them" is ambiguous: does it refer to the nearby noun
"workers" or to the earlier subject "green fields"? Well, only a nitpicker would ever think that it could
refer to "green fields." I have seen many correct answers in GMAT Sentence Correction where a nitpicker
might say that pronoun reference is ambiguous but where a person with common sense would have no
problem.

So, if I had to choose one answer, I would choose B. But I don't think A is so bad.

According to recent studies comparing the nutritional value of meat from wild animals and meat
from domesticated animals, wild animals have less total fat than do livestock fed on grain and more
of a kind of fat they think is good for cardiac health.

A.wild animals have less total fat than do livestock fed on grain and more of a kind of fat they
think is

B.wild animals have less total fat than livestock fed on grain and more of a kind of fat thought to be

C.wild animals have less total fat than that of livestock fed on grain and have more fat of a kind
thought to be

D.total fat of wild animals is less than livestock fed on grain and they have more fat of a kind
thought to be

E.total fat is less in wild animals than that of livestock fed on grain and more of their fat is of a kind
they think is

EXPLANATION BY BOB:

Quote:
Originally Posted by kartikbansal
go for C..

77

Adhamjons SC notes

UZBEKISTAN

'that' is necessary..

"That" is not only unnecessary, but wrong.

When a comparison includes "than that of" the word "that" refers to a specific noun. For example, in the
following sentence:

The population of California is greater than that of New York.

..."that" stands in for "the population." The sentence can also be written:

The population of California is greater than the population of New York.

In choice C of the item under discussion:

wild animals have less total fat than that of livestock fed on grain

...what does "that" stand in for?

The problem with this item is that there is no good answer choice. What is the source? I'm sure it's not a
real GMAT question.

The sentence should be something like this:

According to recent studies comparing the nutritional value of meat from wild animals and that of meat
from domesticated animals, wild animals have less total fat than do livestock fed on grain and more fat of
a kind thought to be good for cardiac health.
Quote:
Originally Posted by BigGameJames
"that" could refer to total fat.

78

Adhamjons SC notes

UZBEKISTAN

The total fat in wild animals is less than that of livestock.

C says: "wild animals have less total fat than that of livestock fed on grain." If "that" stands for "total fat"
then the C means: "wild animals have less total fat than the total fat of livestock fed on grain." To me that
makes no sense. The sentence is trying to compare wild animals with domesticated animals. It should
read: "wild animals have less total fat than do livestock fed on grain."

There is no good answer. I suppose you could say that B is the "least bad," but settling for the "least bad"
is never necessary on the GMAT. There's always a good sentence.

I wish I had stayed out of this discussion. I generally prefer not to comment on bogus questions.

I will try one more time to explain why C is wrong, and then I promise to shut up. A comparison needs
"than that of" or "than those of" only when it is comparing one possession or attribute to another
possession or attribute. Generally the first item in the comparison will either be followed by a
prepositional phrase beginning with "of" or be preceded by a possessive. Examples:

The total mass of all the asteroids is less than that of the Moon.
("That" refers to "mass," which is followed by "of.")

Shanghai's birth rate is lower than that of Western countries.


("That" refers to "birth rate," which is preceded by the possessive "Shanghai's.")

In choice C:

...wild animals have less total fat than that of livestock fed on grain...

"that," I suppose, is meant to refer to "total fat," but "total fat" is not followed by "of" or preceded by a
possessive.

Try googling on "less total fat than" and you will get hundreds of hits, such as these:

Walkers Lites and Potato Heads contain 33% less total fat than Walkers crisps.
"Reduced-fat" ice cream contains at least 25 percent less total fat than the regular product.
79

Adhamjons SC notes

UZBEKISTAN

Body mass indices did not demonstrate that men had less total fat than women.
Peanuts actually have less total fat than most other nuts.
Turkey has less total fat than other meats.
Even the very fattest wild land mammals contain 60-75% less total fat than the average domesticated
animal.
In general, fish and poultry have less total fat than red meats.
Grass-fed beef has a much better fatty acid ration and less total fat than grain-fed animals.
Duck breast meat prepared without the skin (2 grams) has less total fat than chicken (3 grams) and more
fat than turkey (0.5 grams).
Wild deer meat has 44 percent less total fat than beef.
KFC chicken had only slightly less total fat than a Whopper.
The higher muscled-scored animal not only has more total meat but also has less total fat than the lower
muscle-scored animal.
Grass-finished meat has considerably less total fat than grain-fed animals.
Chicken without skin has less saturated fat and less total fat than chicken with skin.
Wild animals not only have less total fat than livestock fed on grain, but more of their fat is of a kind
(omega-3) thought to be good for cardiac health.

(The last example, taken from a New York Times article, is obviously the source of the SC item under
discussion here.)

Try googling on "less total fat than that of" and you will get not one single hit.

9. Because the Earth's crust is more solid there and thus better able to transmit shock waves, an
earthquake of a given magnitude typically devastates an area 100 times greater in the eastern
United States than it does in the West.
(A) of a given magnitude typically devastates an area 100 times greater in the eastern United States
than it does in the West
(B) of a given magnitude will typically devastate 100 times the area if it occurs in the eastern United
States instead of the West
(C) will typically devastate 100 times the area in the eastern United States than one of comparable
magnitude occurring in the West
(D) in the eastern United States will typically devastate an area 100 times greater than will a quake
80

Adhamjons SC notes

UZBEKISTAN

of comparable magnitude occurring in the West 7. It is possible that Native Americans originally
have migrated to the Western Hemisphere over a bridge of land that once existed between Siberia
and Alaska.
(E) that occurs in the eastern United States will typically devastate 100 times more area than if it
occurred with comparable magnitude in the West

EXPLANATION BY BOB:

It is very easy to eliminate C. Remember that "than" can be used only after a comparative. You can say
"an area 100 times greater... than," but you cannot say "100 times the area... than."

The answer is D.

One of Ronald Reagans first acts as President was to rescind President Carters directive that any
chemical banned on medical grounds in the United States be prohibited from sale to other countries.
(A) that any chemical banned on medical grounds in the United States be prohibited from sale to
other countries
(B) that any chemical be prohibited from sale to other countries that was banned on medical
grounds in the United States
(C) prohibiting the sale to other countries of any chemical banned on medical grounds in the United
States
(D) prohibiting that any chemical banned on medical grounds in the United States is sold to other
countries
(E) that any chemical banned in the United States on medical grounds is prohibited from being sold
to other countries.

I know its already dicussed but my question is that isn't the sentence subjuntive by the word - 'directive'?
Should it not be followed by 'that...'. Please Help me clarify on this.

EXPLANATION BY BOB:

When a verb, noun, or adjective of recommendation, requirement, or request is followed by a "that"


clause, the verb in the "that" clause will be in the subjunctive mood. But a verb, noun, or adjective of
81

Adhamjons SC notes

UZBEKISTAN

recommendation, requirement, or request does not have to be followed by a "that" clause.

In the item under discussion, choice A nonsensically states that the directive prohibits chemicals from
doing something. You can't tell chemicals what to do or not to do. You can prohibit the sale of chemicals,
but you can't prohibit chemicals from being sold.

Quote:
Originally Posted by amitg_ind
Thanks Bob for correcting my understanding that ' every subjunctive must be followed by that'.

"Directive" is not a subjunctive. It's a noun that can sometimes require the subjunctive in the following
"that" clause. The subjunctive in choice A is "be."

His studies of ice-polished rocks in his Alpine homeland, far outside the range of present-day
glaciers, led Louis Agassiz in 1837 to propose the concept of an age in which great ice sheets had
existed in now currently temperate areas.
(A) in which great ice sheets had existed in now currently temperate areas
(B) in which great ice sheets existed in what are now temperate areas
(C) when great ice sheets existed where there were areas now temperate
(D) when great ice sheets had existed in current temperate areas
(E) when great ice sheets existed in areas now that are temperate

This had been discussed earlier but I want to clarify my doubt on usage of 'which'. According to me in
GMAT, usage of 'which' without a comma is almost always wrong. However, I see many examples as
exceptions. Can somebody plz clarify the usagee of 'which'?

EXPLANATION BY BOB:

"Which" (not "that") is used in restrictive clauses and without a comma when it follows a preposition.

the stuff of which dreams are made


the book for which I was searching

82

Adhamjons SC notes

UZBEKISTAN

These could be rephrased by moving the preposition to the end and changing "which" to "that":

the stuff that dreams are made of


the book that I was searching for

The latter examples may sound more natural, but old-time grammar police have an unnatural aversion to
prepositions at the end of a sentence.

The increased popularity and availability of televisions has led to the decline of regional dialects,
language variations which originate from diverse ethnic and cultural heritages and perpetuated by
geographic isolation.
(A) which originate from diverse ethnic and cultural heritages and perpetuated
(B) that originated from diverse ethnic and cultural heritages and perpetuated
(C) originated from diverse ethnic and cultural heritages and perpetuated
(D) originating from diverse ethnic and cultural heritages and perpetuated
(E) originating from diverse ethnic and cultural heritages and perpetuating

EXPLANATION BY BOB:

The correct response is D.

B is not parallel. "Originated" is a verb (the simple past). "Perpetuated" is an adjective (past participle).

D is parallel. "Originating" and "perpetuated" are both adjectives. The present participle "originating" is
correct because the variations originate. The past participle "perpetuated" is correct because the variations
don't perpetuate; they are perpetuated.

The European Community exported just under 400 billion in goods in 1998, sixty percent more than
the united states did and nearly twice as much as Japan's export

A .................................................. ...

83

Adhamjons SC notes

UZBEKISTAN

B the united States exported and nearly double what the japanese did
C the United States exported and nearly twice as much as the Japanese did
D what teh United states did and nearly twice as much as japan's exports
E What the United States exported and nearly double the japanese exports

EXPLANATION BY BOB:

For me the best choice is (C).

I split the sentence this way:


The E.C. exported $400 billion in goods in 1998. This amount of goods is 60 percent more than the U.S.
exported. This amount is nearly twice as much as the Japanese exported.

The other choices all compare the amount of goods that the E.C. exported with the actual goods
themselves (not the amount) that the Japanese exported.

1.Believed to originate from a small area on their foreheads, elephants emit low-frequency sounds
that may be used as a secret language to communicate with other members of the herd.
(A) Believed to originate from a small area on their foreheads, elephants emit low-frequency sounds
that may be used
(B) Elephants emit low-frequency sounds that are believed to originate from a small area on their
foreheads, and they may use this
(C) Elephants emit low-frequency sounds, believed to originate from a small area on their
foreheads, that they may use
(D) Originating, it is believed, from a small area on their foreheads; elephants emit low-frequency
sounds they may use
(E) Originating, it is believed, from a small area on their foreheads, low-frequency sounds are
emitted by elephants that may be used

isnt 'they' in choice B refers to subject 'elephant' ........


OA is C

EXPLANATION BY BOB:
84

Adhamjons SC notes

UZBEKISTAN

Problem with B is "this," which is supposed to refer to "sounds."

Best response is C.

The increased popularity and availability of televisions has led to the decline of regional dialects,
language variations which originate from diverse ethnic and cultural heritages and perpetuated by
geographic isolation.
(A) which originate from diverse ethnic and cultural heritages and perpetuated
(B) that originated from diverse ethnic and cultural heritages and perpetuated
(C) originated from diverse ethnic and cultural heritages and perpetuated
(D) originating from diverse ethnic and cultural heritages and perpetuated
(E) originating from diverse ethnic and cultural heritages and perpetuating

OA is D

I am a bit confused about the correct usage of which and comma together.
I think " blah blah blah, which blah blah" is correct.

Also, is it true that WHICH should ALWAYS be preceded by a comma?

I would like some more information about the usage of which and comma.

EXPLANATION BY BOB:

It is misleading and oversimplistic to say that "which" must always be preceded by a comma. The rule is
that a nonrestrictive clause should be set off with commas, but a nonrestrictive clause does not always
begin with "which," and the presence of "which" does not always signal a nonrestrictive clause.

First of all, if the relative pronoun is the object of a preposition, whether the clause is restrictive or
nonrestrictive, you cannot use "that"; you must use "which." Here are a couple of correct sentences from
85

Adhamjons SC notes

UZBEKISTAN

the OG that use "which" after a preposition in a restrictive clause:

El Nino, the periodic abnormal warming of the sea surface off Peru, is a phenomenon in which changes in
the ocean and atmosphere combine to allow the warm water that has accumulated in the western Pacific
to flow back to the east.

As U.S. nuclear attack submarines prowl their familiar haunts deep within the oceans of the world these
days, they increasingly are engaged in missions far different from the tasks for which they were built and
for which their crews were trained over the last forty years.

Second, in contemporary English usage, "which" is very often used where traditionalists would prefer
"that." According to The Little, Brown Handbook:

"Which" can introduce both restrictive and nonrestrictive clauses, but many writers reserve "which" only
for nonrestrictive clauses.

Still, what concerns us here is what GMAT writers think, and if you look at the explanation to OG11 SC7
you will see that choice A is wrong in part because "Which is incorrectly used for a clause that defines,"
and that choice C is wrong in part because "Which incorrectly introduces a restrictive clause." It is worth
noting, however, that both of these wrong choices have other problems; misuse of "which" is not the only
reason to eliminate them.

The fact is that a study of edited prose has shown that about 75 percent of the instances of "which"
introduce restrictive clauses and only about 25 percent introduce nonrestrictive clauses.

different from vs different than


My study shows that different from is the correct idiom, definitely when used with nouns. But im not
sure if it remains correct when used with verbs.

e.g.
The CAT GMAT is different from the pen & paper GMAT
Attempting the CAT GMAT is different than attempting the pen & paper GMAT

86

Adhamjons SC notes

UZBEKISTAN

If vs Whether
I know majority of the sources say, that almost always whether is preferred over if on the GMAT, but
in Ursulas notes it says, that when comparing more than 2 items use if and NOT whether. Is this
correct?

e.g.
John is contemplating if he should get ice cream, frozen yogurt or cookies. OR
John is contemplating whether he should get ice cream, frozen yogurt or cookies.

identical with
Is this the correct idiom or should it be identical to?

had vs had been


I know one is past perfect while the other is past progressive perfect AND that we use past participial with
perfects such as had (e.g. walked, ran etc.) and present participial with progressives/perfect
progressives such as had been (e.g. walking, running etc.) Is this the difference? I just wish to confirm.

EXPLANATION BY BOB:

Quote:
Originally Posted by SidGupta
different from vs different than
My study shows that different from is the correct idiom, definitely when used with nouns. But im not sure
if it remains correct when used with verbs.

e.g.
The CAT GMAT is different from the pen & paper GMAT
Attempting the CAT GMAT is different than attempting the pen & paper GMAT

Your second example is wrong. Should be: "Attempting the CAT GMAT is different from attempting the
pen & paper GMAT."

87

Adhamjons SC notes

UZBEKISTAN

This whole "different than" issue is moderately controversial. People have been saying and writing
"different than" for centuries; it's very common today. I suspect that the GMAT avoids the issue. There is
no instance of "different than" in the Official Guides, for example.
Quote:
Originally Posted by SidGupta
If vs Whether
I know majority of the sources say, that almost always whether is preferred over if on the GMAT, but
in Ursulas notes it says, that when comparing more than 2 items use if and NOT whether. Is this
correct?

e.g.
John is contemplating if he should get ice cream, frozen yogurt or cookies. OR
John is contemplating whether he should get ice cream, frozen yogurt or cookies.

To express alternatives, "whether" is preferred. It makes no difference how many alternatives there are.
Quote:
Originally Posted by SidGupta
identical with
Is this the correct idiom or should it be identical to?

"Identical with" is more common, but both "identical with" and "identical to" are fully acceptable.
Quote:
Originally Posted by SidGupta
had vs had been
I know one is past perfect while the other is past progressive perfect AND that we use past participial with
perfects such as had (e.g. walked, ran etc.) and present participial with progressives/perfect
progressives such as had been (e.g. walking, running etc.) Is this the difference? I just wish to confirm.

Past perfect ("had walked") is generally used to describe an action completed before another action in the
past. Past perfect progressive ("had been walked") is generally used to described an action that started
88

Adhamjons SC notes

UZBEKISTAN

earlier and continued up to a time in the past. Examples:

He was tired all afternoon because he had walked five miles in the morning.
He had been walking for an hour when it started to rain.

Note, however, that there are many verbs that are rarely used in progressive forms. For example: believe,
forget, imagine, know, want, like, prefer, have, possess, hear, see, smell, taste, be. I can't imagine ever
saying, for example, "had been being."

10. Climatic shifts are so gradual as to be indistinguishable at first from ordinary fluctuations in the
weather.

(A) so gradual as to be indistinguishable


(B) so gradual they can be indistinguishable
(C) so gradual that they are unable to be distinguished
(D) gradual enough not to be distinguishable
(E) gradual enough so that one cannot distinguish them

I though So as to be is for singular subjects!


Answ is A I have chosen C

EXPLANATION BY BOB:

Problem with (C) is "unable to be distinguished." Never write "able to be X-ed" or "unable to be X-ed."
To be X-ed is not an ability that one can or cannot possess. This is the worst possible passive construction.

Sixty-five milion years ago, according to some scientists, an asteroid bigger than Mount Everest
slammed into North America, which, causing plant and animal extinctions, marks the end of the
geologic era known as the Cretaceous period.

A: ...
B: which caused the plant and animal extinctions marking
89

Adhamjons SC notes

UZBEKISTAN

C: and causing plant and animal extinctions that mark


D: an event that caused plant and animal extinctions, and it marks
E: an event that caused the plant and animal extinctions that mark

EXPLANATION BY BOB:

In D, "it" is ambiguous. Is the antecedent "asteroid," "Mount Everest," "North America," or "event"? I see
nothing wrong with E.

Despite the growing number of people who purchase plane tickets online, airline executives are
convinced that, just as one-third of bank customers still prefer human tellers to automatic teller
machines, many travelers will still use travel agents.

A. growing number of people who purchase plane tickets online, airline executives are convinced
that, just as one-third of bank customers still prefer human tellers to automatic teller machines,
many travelers will
B. growing number of people who purchase plane tickets online, airline executives are convinced,
just as one-third of bank customers still prefer human tellers to automatic teller machines, that
many travelers would
C. growing number of people purchasing plane tickets online, airline executives are convinced, just
as one-third of bank customers still prefer human tellers as compared to automatic teller machines,
many travelers will
D. fact that the number of people purchasing plane tickets online is growing, airline executives are
convinced, just as one-third of bank customers still prefer human tellers as compared to automatic
teller machines, that many travelers would
E. fact that the number of people who purchase plane tickets online are growing, airline executives
are convinced that, just as one-third of bank customers still prefer human tellers compared with
automatic teller machines, many travelers would

EXPLANATION BY BOB:

90

Adhamjons SC notes

UZBEKISTAN

For me the best choice is (A).

We can quickly eliminate (C), (D), and (E) because of the unidiomatic "prefer X as compared to Y" and
"prefer X compared with Y." Only (A) and (B) use the correct idiom "prefer X to Y."

Between (A) and (B), the former is to be preferred because of the placement of "that." The bank
customers' preference is part of what the airline executives are convinced of, and so "that" should come
before that clause.

Hi all,I wish I had found this forum earlier! My GMAT is on Wednesday! =(A question about parallel
construction. Which sentence is correct, as far as GMAT grammar is concerned?A) She runs faster than
I.B) She runs faster than do I.C) She runs faster than I do.I am getting conflicting answers according to the
Princeton and the Barron's books respectively.Thank you!

EXPLANATION BY BOB:
She runs faster than I.
She runs faster than do I.
She runs faster than I do.

All three are grammatically correct. The first is best because it is the most concise. There is no need for
the "do" in this case because there subject pronoun "I" is clearly being compared with the subject "she." A
verb such as "do" or "does" or "is" is necessary after "than" only when there are both a subject and an
object before.

Example:
"Martin likes football more than his wife." Ambiguous. It is not clear whether "wife" is being compared
with "Martin" or "football."
Should be either:
"Martin likes football more than does his wife."
Or:
"Martin likes football more than his wife does."
[Both of these compare "wife" with "Martin."]
91

Adhamjons SC notes

UZBEKISTAN

Or:
"Martin likes football more than he likes his wife."
Or:
"Martin likes football more than he does his wife."
[These last two versions compare "wife" with "football."]

No verb is needed after "than" when there is no object. Example:


"Martin drives faster than his wife."
You could write:
"Martin drives faster than his wife does."
Or:
"Martin drives faster than does his wife."
But the "does" is not really necessary because there is only one noun ("Martin") with which "wife" could
be compared.

In any case, "she runs faster than me" is not grammatically correct. "Than" is not a preposition and does
not require the object pronoun. "She runs faster than I" is short for "she runs faster than I do."

Quote:
Originally Posted by msinh
Bob,
I was going through few of the Kaplan's explanation and got confused again. Look at these sentences:

The Boston office contributes less to total national sales than any other U.S. branch.
And Kaplan says above is wrong. According to Kaplan, comparison is between what the Boston office
contributes to what any other U.S. branch contributes. It further says that in GMAT english, the most
elegant way to fix this is to insert does after U.S. branch. So corrected sentences will be:
The Boston office contributes less to total national sales than any other U.S. branch does.

Kaplan is partly right and partly wrong. The original version of the sentence is ambiguous: it is not
immediately evident whether "any other U.S. branch" is being compared with the subject "the Boston
office" or the object "total national sales." So the "does" necessary.

Kaplan is wrong, however, in saying that the most "elegant" position for "does" is at the end. You can also
92

Adhamjons SC notes

UZBEKISTAN

put "does" in front of "any other branch":

The Boston office contributes less to total national sales than does any other U.S. branch.

In fact, traditionally it was necessary to invert the subject and verb after "than," and that's what the GMAT
usually does. Today, however, it is more common in everyday speech to put "does" at the end. Both
positions are acceptable, and the GMAT will never force you to choose between them.

Quote:
Originally Posted by msinh
Please let us know what to follow in actual GMAT test. I couldn't get any OG example based on this
concept.

Here are some examples from the OG 10th edition. First, there are many examples of correct sentences in
which the verb is omitted after "than":

5. Carbon-14 dating reveals that the megalithic monuments in Brittany are nearly 2,000 years older than
any of their supposed Mediterranean predecessors.

Could just as well be:


...older than are any of the supposed Mediterranean predecessors.
Or:
...older than any of the supposed Mediterranean predecessors are.

46. A study commissioned by the Department of Agriculture showed that if calves exercise and associate
with other calves, they require less medication and gain weight more quickly than those raised in
confinement.

Could just as well be:


...more quickly than do those raised in confinement.
Or:
...more quickly than those raised in confinement do.

93

Adhamjons SC notes

UZBEKISTAN

118. The question of whether to divest themselves of stock in companies that do business in South Africa
is particularly troublesome for the nation's 116 private Black colleges because their economic bases are
often more fragile than those of most predominantly White colleges.

Could just as well be:


...more fragile than are those of most predominantly White colleges.
Or:
...more fragile than those of most predominantly White colleges are.

149. Last year, land values in most parts of the pinelands rose almost as fast as, and in some parts even
faster than, those outside the pinelands.

Could just as well be:


...as fast as, and in some parts even faster than, did those outside the pinelands.
Or:
...as fast as, and in some parts even faster than, those outside the pinelands did.

268. Ranked as one of the most important of Europe's young playwrights, Franz Xaver Kroetz has written
forty plays; his works--translated into over thirty languages--are produced more often than those of any
other contemporary German dramatist.

Could just as well be:


...more often than are those of any other contemporary German dramatist.
Or:
...more often than those of any other contemporary German dramatist are.

And here are two examples of correct sentences in which the verb precedes the subject after "than":

199. Because the Earth's crust is more solid there and thus better able to transmit shock waves, an
earthquake in the eastern United States will typically devastate an area 100 times greater than will a
quake of comparable magnitude occurring in the West.

94

Adhamjons SC notes

UZBEKISTAN

224. Inuits of the Bering Sea were isolated from contact with Europeans longer than were Aleuts or
Inuits of the North Pacific and northern Alaska.

On the other hand, here is one correct sentence in which the verb follows the noun after "than":

158. Despite having only 5 percent of the world's population, United States citizens consume 28 percent of
its nonrenewable resources, drive more than one-third of its automobiles, and use 21 times more water
per capita than Europeans do.

Quote:
Originally Posted by msinh
Kaplan says ... that in GMAT english, the most elegant way to fix this is to insert does after U.S. branch.

What does Kaplan say about the following 36 sentences taken from OG 10th edition Critical Reasoning?
Are these all inelegant?

33. Rural households have more purchasing power than do urban or suburban households at the same
income level...

33. (A) The average rural household includes more people than does the average urban or suburban
household.

33. (B) Rural households have lower food and housing costs than do either urban or suburban households.

33. (C) Suburban households generally have more purchasing power than do either rural or urban
households.

48. A researcher discovered that people who have low levels of immune-system activity tend to score
much lower on tests of mental health than do people with normal or high immune-system activity.

51. (B) Determining whether classical guitarists make their strings go dead faster than do folk guitarists

63. However, a recent study found that top managers used intuition significantly more than did most
middle-or lower-level managers.
95

Adhamjons SC notes

UZBEKISTAN

64. (D) Domestic "mini-mills" consistently produce better grades of steel than do the big American mills.

67. (C) The average citizen of Town S spends less time reading newspapers than does the average citizen
of Town T.

70. (D) Shelby Industries pays its employees, on average, ten percent more than does Jones Industries.

80. (C) Domestic production of illegal drugs increased at a higher rate than did the entry of such drugs
into the country.

81. (A) in 1987 smugglers of illegal drugs, as a group, had significantly more funds at their disposal than
did the country's customs agents

84. (D) Two-year community colleges devote more resources than do other colleges to attracting those
students especially affected by economic slowdowns.

107. (D) Between 1964 and 1978 the United States consistently spent a larger percentage of its GNP on
research and development than did Japan.

127. (E) Service companies make up the majority of small businesses and they generally employ more
employees per dollar of revenues than do retail or manufacturing businesses.

128. (D) The particular methods currently used by European painters could require less artistic skill than
do methods used by painters in other parts of the world.

137. A study of marital relationships in which one partner's sleeping and waking cycles differ from those
of the other partner reveals that such couples share fewer activities with each other and have more violent
arguments than do couples in a relationship in which both partners follow the same sleeping and waking
patterns.

139. (E) knowledge that a personal acquaintance is unemployed generates more fear of losing one's job
than does knowledge of unemployment statistics

96

Adhamjons SC notes

UZBEKISTAN

143. (C) Experienced users of Microton computers command much higher salaries than do prospective
employees who have no experience in the use of computers.

145. (D) Nursing homes built prior to 1985 generally had fewer beds than did nursing homes
built between 1985 and 1988.

156. According to records maintained by each company, the employees of Company O had fewer jobrelated accidents last year than did the employees of Company P.

156. (A) Company P manufactures products that are more hazardous for workers to produce than does
Company O.

156. (B) Company P holds more safety inspections than does Company O.

156. (C) Company P maintains a more modern infirmary than does Company O.

156. (D) Company O paid more for new job-related medical claims than did Company P.

156. (E) Company P provides more types of health-care benefits than does Company O.

157. (A) The employees of Company P lost more time at work due to job-related accidents than did the
employees of Company O.

157. (B) Company P considered more types of accidents to be job-related than did Company O.

158. (B) EFCO keyboards are no more expensive to manufacture than are standard keyboards and require
less frequent repair than do standard keyboards.

159. Afterward, this group showed lower cognitive abilities than did the other half of the subjectsthe
control groupwho did not consume the sweetener.

161. (D) A female rat that has given birth shows maternal behavior toward rat pups that she did not bear
more quickly than does a female rat that has never given birth.

97

Adhamjons SC notes

UZBEKISTAN

171. (E) Drivers on Maryland highways exceeded the speed limit more often than did drivers on other
state highways not covered in the report.

173. They will thus burn significantly fewer calories at the new weight than do people whose weight is
normally at that level.

173. (A) Relatively few very overweight people who have dieted down to a new weight tend to continue
to consume substantially fewer calories than do people whose normal weight is at that level.

187. (A) The Highway Traffic Safety Institute report listed many cars in other classes that had more
injuries per accident than did the PZ 1000.

189. (B) Elderly people who practice a religion appear to experience less anxiety at the prospect of dying
than do other people.

According to a survey of graduating medical students conducted by the Association of American


Medical Colleges, minority graduates are nearly four times more likely than are other graduates in
planning to practice in socioeconomically deprived areas.

(A) minority graduates are nearly four times more likely than are other graduates in planning to
practice
(B) minority graduates are nearly four times more likely than other graduates who plan on
practicing
(C) minority graduates are nearly four times as likely as other graduates to plan on practicing
(D) it is nearly four times more likely that minority graduates rather than other graduates will plan
to practice
(E) it is nearly four times as likely for minority graduates than other graduates to plan to practice

EXPLANATION BY BOB:

Quote:

98

Adhamjons SC notes

UZBEKISTAN

Originally Posted by shanmukh


i do not see any wrong with A
"likely ... in planning" is unidiomatic.
I think the correct response is C.

A 1972 agreement between Canada and the United States reduced the amount of phosphates that
municipalities had been allowed to dump into the Great Lakes.

A) reduced the amount of phosphates that municipalities had been allowed to dump

B) reduced the phosphate amount that municipalities had been dumping

C) reduces the phosphate amount that municipalities have been allowed to dump

D) reduced the amount of phosphates that municipalities are allowed to dump

E) reduces the amount of phosphates allowed for dumping by municipalities

Why A is wrong? Why can't this sentence be read as - 1972 agreement reduced the amount, the dumping
amount which was allowed prior to 1972 agreement. What I mean to say - e.g. before 1972 , allowable
quantity for dumping was X, 1972 agreement reduced this amount.

OG says D is correct - Since the dumping continues into the present, the past perfect verb had been
allowed should instead be the present are allowed

I think D changes meaning. How one can assume that dumping still continues- there could be another
agreement, which might have banned phosphate dumping altogether.

EXPLANATION BY BOB:

I agree with D. An agreement cannot reduce the amount that had been allowed. That previous amount
cannot be changed. There is an allowable amount. That amount was bigger before the agreement, and has
been reduced as a result of the agreement.
99

Adhamjons SC notes

UZBEKISTAN

Analogy: Let's say the legislature has changed the speed limit. Which sounds better?

1) The legislature increased the maximum speed that one is allowed to drive.
2) The legislature increased the maximum speed that one had been allowed to drive.

I'll go with 1

EXPLANATION BY ERIN:

Great discussions here. And special thanks to 800bob for sharing his great knowledge of the GMAT. It's
nice to have another teacher here.

If I may be allowed to jump in, I'd like to add my two cents. First, I should note that while you should not
change the meaning of A, you will notice in some cases that the literal meaning of A is often illogical; in
such cases, you may deviate from the original meaning of A. I think it would be more accurate to say that
you should not change the intended meaning of A. Of course we could get into some philosophical
discussions of intended meaning, but for the sake of raising your GMAT score, I think it's most productive
to try to play along with GMAT.

Second, to address this question, it doesn't make sense to use the past tense to refer to the result of the
agreement; if you use the past tense, you are saying that the US and Canada agreed to retroactively
change the amount of phosphates that could be dumped, which would in all likelihood have the effect of
creating a lot of lawbreakers.

Since the vast majority of changes to laws are made such that the

changes take place at some point in the future, you can safely assume that this is the case here as well.

Using the present tense simply means that the new law is in effect right now.

1. After this years record-shattering January performance in Madison Square Garden, the
ensemble were touted as the countrys best new group in decades; no critic or reviewer had
anything but praise for the young musicians.
100

Adhamjons SC notes

UZBEKISTAN

(A) the ensemble were touted as the countrys

(B) the ensemble was touted as the countrys

(C) the country touted the ensemble like the

(D) touting the ensemble as the countrys

(E) they were touting the ensemble as the countrys

2. Alaska regularly deposits some of its profits from the sale of oil into a special fund, with the
intention to sustain the states economy after the exhaustion of its oil reserves.

(A) fund, with the intention to sustain the states economy after the exhaustion of its oil reserves

(B) fund, the intention of which is to sustain the states economy after they have exhausted their oil
reserves

(C) fund intended to sustain the states economy after oil reserves are exhausted

(D) fund intended to sustain the states economy after exhausting its oil reserves

(E) fund that they intend to sustain the states economy after oil reserves are exhausted

3. Camille Claudet worked continuously through the 1880s and early 1890s with the sculptor
Auguste Rodin; since there are very few signed works of hers, the inescapable conclusion seems to
be one of Claudet conceiving and executing part of Rodins enormous production of that period.

(A) inescapable conclusion seems to be one of Claudet conceiving and executing part of Rodins
enormous production of that period

(B) conclusion of Claudet conceiving and executing part of Rodins enormous production of that
101

Adhamjons SC notes

UZBEKISTAN

period seems inescapable

(C) conclusion seems inescapable that part of Rodins enormous production of that period was
conceived and executed by Claudet

(D) conclusion of part of Rodins enormous production of that period having been conceived and
executed by Claudet seems inescapable

(E) seemingly inescapable conclusion is that Claudet would have conceived and executed part of
Rodins enormous production of that period

4. Canadian scientists have calculated that one human being should be struck every nine years by a
meteorite, while each year sixteen buildings can be expected to sustain damage from such objects.

(A) one human being should be struck every nine years by a meteorite

(B) a human being should be struck by a meteorite once in every nine years

(C) a meteorite will strike one human being once in every nine years

(D) every nine years a human being will be struck by a meteorite

(E) every nine years a human being should be struck by a meteorite

5. Certain pesticides can become ineffective if used repeatedly in the same place; one reason is
suggested by the finding that there are much larger populations of pesticide-degrading microbes in
soils with a relatively long history of pesticide use than in soils that are free of such chemicals.

(A) Certain pesticides can become ineffective if used repeatedly in the same place; one reason is
suggested by the finding that there are much larger populations of pesticide-degrading microbes in
soils with a relatively long history of pesticide use than in soils that are free of such chemicals.

(B) If used repeatedly in the same place, one reason that certain pesticides can become ineffective is
102

Adhamjons SC notes

UZBEKISTAN

suggested by the finding that there are much larger populations of pesticide-degrading microbes in
soils with a relatively long history of pesticide use than in soils that are free of such chemicals.

(C) If used repeatedly in the same place, one reason certain pesticides can become ineffective is
suggested by the finding that much larger populations of pesticide-degrading microbes are found in
soils with a relatively long history of pesticide use than those that are free of such chemicals.

(D) The finding that there are much larger populations of pesticide-degrading microbes in soils with
a relatively long history of pesticide use than in soils that are free of such chemicals is suggestive of
one reason, if used repeatedly in the same place, certain pesticides can become ineffective.

(E) The finding of much larger populations of pesticide-degrading microbes in soils with a relatively
long history of pesticide use than in those that are free of such chemicals suggests one reason certain
pesticides can become ineffective if used repeatedly in the same place.

EXPLANATION BY BOB:

The correct responses are B, C, C, D, A.

The point of this exercise was to debunk the shibboleth that active is always better than passive. In every
one of these items, the correct response uses a passive construction and at least one of the incorrect
responses uses the active.

1. After this years record-shattering January performance in Madison Square Garden, the ensemble were
touted as the countrys best new group in decades; no critic or reviewer had anything but praise for the
young musicians.
(A) the ensemble were touted as the countrys
(B) the ensemble was touted as the countrys (passive -- correct)
(C) the country touted the ensemble like the (active -- incorrect)
(D) touting the ensemble as the countrys (active -- incorrect)
(E) they were touting the ensemble as the countrys (active -- incorrect)

2. Alaska regularly deposits some of its profits from the sale of oil into a special fund, with the intention to
sustain the states economy after the exhaustion of its oil reserves.
103

Adhamjons SC notes

UZBEKISTAN

(A) fund, with the intention to sustain the states economy after the exhaustion of its oil reserves
(B) fund, the intention of which is to sustain the states economy after they have exhausted their oil
reserves (active -- incorrect)
(C) fund intended to sustain the states economy after oil reserves are exhausted (passive -- correct)
(D) fund intended to sustain the states economy after exhausting its oil reserves (active -- incorrect)
(E) fund that they intend to sustain the states economy after oil reserves are exhausted

3. Camille Claudet worked continuously through the 1880s and early 1890s with the sculptor Auguste
Rodin; since there are very few signed works of hers, the inescapable conclusion seems to be one of
Claudet conceiving and executing part of Rodins enormous production of that period.
(A) inescapable conclusion seems to be one of Claudet conceiving and executing part of Rodins
enormous production of that period (active -- incorrect)
(B) conclusion of Claudet conceiving and executing part of Rodins enormous production of that
period seems inescapable (active -- incorrect)
(C) conclusion seems inescapable that part of Rodins enormous production of that period was
conceived and executed by Claudet (passive -- correct)
(D) conclusion of part of Rodins enormous production of that period having been conceived and executed
by Claudet seems inescapable
(E) seemingly inescapable conclusion is that Claudet would have conceived and executed part of
Rodins enormous production of that period (active -- incorrect)

4. Canadian scientists have calculated that one human being should be struck every nine years by a
meteorite, while each year sixteen buildings can be expected to sustain damage from such objects.
(A) one human being should be struck every nine years by a meteorite
(B) a human being should be struck by a meteorite once in every nine years
(C) a meteorite will strike one human being once in every nine years (active -- incorrect)
(D) every nine years a human being will be struck by a meteorite (passive -- correct)
(E) every nine years a human being should be struck by a meteorite

5. Certain pesticides can become ineffective if used repeatedly in the same place; one reason is suggested
by the finding that there are much larger populations of pesticide-degrading microbes in soils with a
relatively long history of pesticide use than in soils that are free of such chemicals.
(A) Certain pesticides can become ineffective if used repeatedly in the same place; one reason is
suggested by the finding that there are much larger populations of pesticide-degrading microbes in soils
104

Adhamjons SC notes

UZBEKISTAN

with a relatively long history of pesticide use than in soils that are free of such chemicals. (passive -correct)
(B) If used repeatedly in the same place, one reason that certain pesticides can become ineffective is
suggested by the finding that there are much larger populations of pesticide-degrading microbes in soils
with a relatively long history of pesticide use than in soils that are free of such chemicals.
(C) If used repeatedly in the same place, one reason certain pesticides can become ineffective is suggested
by the finding that much larger populations of pesticide-degrading microbes are found in soils with a
relatively long history of pesticide use than those that are free of such chemicals.
(D) The finding that there are much larger populations of pesticide-degrading microbes in soils with a
relatively long history of pesticide use than in soils that are free of such chemicals is suggestive of one
reason, if used repeatedly in the same place, certain pesticides can become ineffective.
(E) The finding of much larger populations of pesticide-degrading microbes in soils with a relatively long
history of pesticide use than in those that are free of such chemicals suggests one reason certain
pesticides can become ineffective if used repeatedly in the same place. (active -- incorrect)

I am still looking for an example of a realistic GMAT Sentence Completion in which identifying the
correct answer hinges on preferring active to passive.
Quote:
Originally Posted by sunilbhat
I checked other threads for the same questions and found that answer for Q 2 is C. But I disagree with the
answer because the highlighted part of the sentence doesnt tell whose oil reserves are exhausted.

All choices other than C have outright errors. A has the unidiomatic "intention to." B and E both use the
plural pronoun "they" to refer to the singular "Alaska." D is wrong because "fund, intended to sustain the
state's economy after exhausting its oil reserves" means that the fund will sustain the economy after the
fund exhausts its oil reserves. In fact, I think that the reference of "its" in both A and D is ambiguous. It is
Alaska's oil reserves, the fund's oil reserves, or the economy's oil reserves?

The fact of some fraternal twins resembling each other greatly and others looking quite dissimilar
highlights an interesting and often overlooked feature of fraternal-twin pairs, namely they vary
considerably on a spectrum of genetic relatedness.

A. The fact of some fraternal twins resembling each other greatly and others looking quite
105

Adhamjons SC notes

UZBEKISTAN

dissimilar highlights an interesting and often overlooked feature of fraternal-twin pairs, namely
they vary considerably

B. That some fraternal twins resemble each other greatly while others look quite dissimilar
highlights an interesting and often overlooked feature of fraternal-twin pairs, namely that they vary
considerably

C. With some fraternal twins resembling each other greatly and others looking quite dissimilar, it
highlights an interesting and often overlooked feature of fraternal-twin pairs, namely considerable
variation

D. With some fraternal twins resembling each other greatly and others looking quite dissimilar, it is
a fact that highlights an interesting and often overlooked feature of fraternal-twin pairs, namely a
considerable variation

E. Because some fraternal twins resemble each other greatly and others look quite dissimilar, this
fact highlights an interesting and often overlooked feature of fraternal-twin pairs, namely they vary
considerably

SPOILER: OA: B
The answer is an uncompleted sentence?
I can't find the appropriate verb in it. Could anyone please explain?

EXPLANATION BY BOB:

This is a case in which the subject is a whole clause: "that some fraternal twins resemble each other
greatly while others look quite dissimilar." That is the thing that is doing the highlighting.

Quote:
Originally Posted by maverick312
can any one explain whats wrong with A?

106

Adhamjons SC notes

UZBEKISTAN

"The fact of + noun + gerund..." - unidiomatic.


"The fact that + noun + verb..." - better.
"That + noun + verb...." - best.

Example:
"The fact of the GMAT being hard cannot be denied." - unidiomatic and ugly.
"The fact that the GMAT is hard cannot be denied." - OK, but wordy.
"That the GMAT is hard cannot be denied." - best.

For many travelers, charter vacations often turn out to cost considerably more than they originally
seemed.

(A) they originally seemed


(B) they originally seem to
(C) they seemingly would cost originally
(D) it seemed originally
(E) it originally seemed they would

EXPLANATION BY BOB:

Quote/ FROM SENTENCECORRECTION.COM

I vote for E.
I think only E is grammatically correct.

a. they originally seemed to


b. they originally seemed to
c. they seemingly? would cost originally?
d. it seemed originally that they would
e. it originally seemed (that) they would (cost).

This is a good start. For instance, B would need to be in past tense, seemed. In fact, I often teach GMAT
and TOEFL students that the words original (and its derivations) and first usually require past tense.
107

Adhamjons SC notes

UZBEKISTAN

Furthermore, the "past future" of would in E is more precise than the simple past in A (or that B is lacking,
in case you're going to ask if B would be correct if we changed seem to seemed). Since we're talking about
something we'd learn after a certain point in the past, would is better.

For example:
This Acura is a lot better than I thought it would be.
is better than
This Acura is a lot better than I thought it was.
In the first example, we are saying that something turned out to be true. In the second example, we are
saying that we were not aware of a fact that was true at that time.

And in the charter vacations question, there is no fact that the travelers were unaware of when they
purchased the charter vacation--after the original purchase of the charter vacation (and probably toward
the end of the vacation), the charter vacation turned out to be more costly than they had at first believed it
would be.

EXPLANATION BY BOB:
The answer is E.

At a recent session, the French government has decided that Paris needs a second, larger opera
house to complement the famous Paris Opera.

(A) has decided that Paris needs

(B) decided that Paris needs

(C) has decided that Paris will need

(D) decided that Paris has a need of


108

Adhamjons SC notes

UZBEKISTAN

(E) has decided that Paris needed


SPOILER: B

Guys I need your help with this one:

In my opinion the answer should have read : "decided that Paris need..."

I looked for examples on the internet and found that the subjunctive is needed with the verb "decide".

EXPLANATION BY BOB:

Decide" does not usually take the subjunctive. It depends on how it is used. Examples:

The court decided that code is protected by the free speech.


The court decided that the membership of the Appointing Committee for this Chair be as follows...

In the first example the court's decision is a statement of fact, hence the indicative. In the second example
the court's decision is a mandate, hence the subjunctive. The indicative is far more common after "decide."

There are other verbs that sometimes take the subjunctive and sometimes take the indicative, depending
on the meaning. Examples:

The teacher suggested that the student write more clearly.


The teacher suggested that the student writes poorly.

In the first example, the teacher's suggestion is a recommendation, hence the subjunctive. In the second
example the teacher's suggestion is a statement of fact, hence the indicative.

109

Adhamjons SC notes

UZBEKISTAN

Rivaling the pyramids of Egypt or even the ancient cities of the Maya as an achievement, the army
of terra-cotta warriors created to protect Qin Shi Huang, Chinas first emperor, in his afterlife is
more than 2,000 years old and took 700,000 artisans more than 36 years to complete them.
A. took 700,000 artisans more than 36 years to complete them
B. took 700,000 artisans more than 36 years to complete it
C. took 700,000 artisans more than 36 years to complete
D. 700,000 artisans took more than 36 years to complete
E. to complete them too 700,000 artisans more than 36 years

EXPLANATION BY BOB:

OA is wrong. Correct response is C.

... army ... is more than 2,000 years old and took ... more than 36 years to complete.

Choice B is wrong because of the superfluous "it." Examples

Gould's most recent book, "The Structure of Evolutionary Theory" (2002), is a 1433-page opus that took
him more than 20 years to complete.

Susanna Clarke admits that her first novel took her more than 10 years to write.

The letter, of just 250 or so Chinese characters, took her more than two hours to compose.

'contrast with/to/between' AND 'compare to/with' - THE FINAL WORD

Im sorry, i know this query has been posted MULTIPLE times. I searched and went through each thread,
but STILL didnt get a satisfactory answer.

110

Adhamjons SC notes

UZBEKISTAN

Ive gone through the PR, the kaplan 2006, the 11th ed OG, ursula's notes and the Kaplan 800 and ALL
have varying definitions/explanations.

CONTRAST
-----------

For e.g. The K800 states it almost as a RULE that 'in contrast to' is the correct idiom, while many sources
say that 'contrast with' is preferred. OR does the 'in' change this preference.

So it should be 'contrast A with B', but 'A in contrast to B' ? Im so confused ...

I googled and i got the following from thefreedicitonary.com:

Usage Note: The noun contrast may be followed by between, with, or to: There is a sharp contrast
between his earlier and later works. In contrast with (or less frequently, to) his early works, the later plays
are dark and forbidding. When contrast is used as a transitive verb, both with and to may follow, though
with is more common: Most scholars contrast the light comedies of his early career with (or to) the dark
comedies that were written late in his life.

So, now the K800s claim that 'in contrast to' is the correct idiom goes in the bin?

COMPARE
---------

This only gets tougher. When LIKE things are discussed, then use 'compare with' and when UNLIKE
things are discussed use 'compare to' (specially when stressing similarities, apparently). But, how do you
decide WHAT IS LIKE and what is not? What if neither similarity nor difference is being stressed, but
rather who/what is better?

Is 'I' and 'YOU' like or unlike?


I am better compared with/to you?

Going by my ear, 'to' sounds better. By definition, im not sure whether similarities or differences are being
111

Adhamjons SC notes

UZBEKISTAN

discussed here OR whether 'I' and 'YOU' are like or unlike.

What about 'ice cream' and 'brocolli'


Broccoli is good for you compared with/to ice cream.

To me its clear that ice cream and brocolli are UNLIKE items, so the above DEFINITELY should be
'compare to', but 800score has quoted is an example of 'compare with'. Why?

What about 'Jimmy Steward' and 'Gregory Peck'? Like or unlike?


Gregory Peck was a better actor compared to/with Jimmy Stewart.

My ear tells me nothing. Probably similarities are being stressed here or maybe theyre not, i dont know.
Are the two actors LIKE or UNLIKE? I DONT KNOW!

thefreedicitonary.com usage is as confusing:

Usage Note: Compare usually takes the preposition to when it refers to the activity of describing the
resemblances between unlike things: He compared her to a summer day. Scientists sometimes compare the
human brain to a computer. It takes with when it refers to the act of examining two like things in order to
discern their similarities or differences: The police compared the forged signature with the original. The
committee will have to compare the Senate's version of the bill with the version that was passed by the
House. When compare is used to mean "to liken (one) with another," with is traditionally held to be the
correct preposition: That little bauble is not to be compared with (not to) this enormous jewel. But to is
frequently used in this context and is not incorrect.

Im giving the test in 3 days, LORD someone solve this one for me. Please

EXPLANATION BY BOB:

Here is the definitive answer to the question about in contrast withversusin contrast to. OG11, p.669,
explanation to question 16:

"The intent of the sentence is to contrast the honeybee and the yellow jacket. Correct idioms for such a
112

Adhamjons SC notes

UZBEKISTAN

contrast include: in contrast with x, y; in contrast to x, y; unlike x, y."

In short, both forms are correct.

And here's the definitive answer to the question about compare with versus compare to:

THERE IS NO DIFFERENCE. THEY ARE BOTH CORRECT, AND THEY ARE


INTERCHANGEABLE.

A study of OG explanations has revealed that GMAT writers use the two forms interchangeably. Each
form is sometimes used to point out similarities and sometimes used to point out differences. If there ever
was a difference between compare with and compare to, that difference is no longer observed by even the
best writers today. I have never seen a real GMAT SC item that tested the difference between compare
with and compare to.

But neither form is correct after better. After a comparative adjective or adverb, you must use than. Not
"better actor compared with" or "better actor compared to," but "better actor than."

In recent years cattle breeders have increasingly used crossbreeding, in part that their steers should
acquire certain characteristics and partly because crossbreeding is said to provide bybrid vigor.
A. B. C. D. partly because certain characteristics should be acquired by their steers
E. partly to acquire certain characteristics in their steers
SPOILER:
OA is E

The official explanation for D says - D awkwardly and illogically shifts to the passive voice...the steers,
however, are not agents in the aquisition. But why? If the steers are not agents, then who would be ? The

113

Adhamjons SC notes

UZBEKISTAN

breeders ? I am confused about the latter half explanation.


Who can give me some tips ?

EXPLANATION BY BOB:

Quote:
Originally Posted by ShalomFang
But why? If the steers are not agents, then who would be ? The breeders ?

Exactly. "Agent" means one who produces an effect. It's not the steers that produce the effect; it's the
breeders who make the steers acquire certain characteristics.

Car owners who inflate their tires properly can substantially boost their vehicles fuel efficiency, since
the increase in car-road friction can waste up to five percent of car fuel by underinflation.
(A) Car owners who inflate their tires properly can substantially boost their vehicles fuel efficiency,
since the increase in car-road friction can waste up to five percent of car fuel by underinflation.
(B) Because the underinflation of tires can waste up to five percent of a cars fuel by increasing carroad friction, car owners can substantially boost their vehicles fuel efficiency by properly inflating
the tires.
(C) Their vehicles fuel efficiency is substantially boosted by car owners through the proper
inflation of tires that, when underinflated, can waste up to five percent of car fuel by an increase in
car-road friction.
(D) The proper inflation of tires by car owners, due to the fact that underinflation can waste up to
five percent of a cars fuel by the increase of car-road friction, can substantially boost their fuel
efficiency.
(E) Because up to five percent of a cars fuel are wasted through the increases in car-road friction
when the tires are underinflated, car owners properly inflating tires can substantially boost their
fuel efficiency.

EXPLANATION BY BOB:
114

Adhamjons SC notes

UZBEKISTAN

As written above, E cannot be the correct answer. Verb does not agree with subject in "five percent of a
car's fuel are wasted," and "cars owners properly inflating tires" cannot be a subject. Correct answer must
be one of the choices not shown.

PREFERED ANSWER IS B

Dr. Sayres lecture recounted several little-know episodes in the relations between nations that
illustrates what is wrong with alliances and treaties that do not have popular support.

(A) relations between nations that illustrates

(B) relation of one nation with another that illustrates

(C) relations between nations that illustrate

(D) relation of one nation with another and illustrate

(E) relations of nations that illustrates

OA-B

EXPLANATION BY BOB:

C is the only possible answer. "That" refers to "episodes," which is plural and therefore requires the plural
verb "illustrate."

It is an oversimplification to say that one cannot use "between" when more than two parties are involved.
"Among" is indeed usually preferred in that case:

The money was divided between the two sons.


The money was divided among the three children.

115

Adhamjons SC notes

UZBEKISTAN

However, "between" is used when speaking about a relationship involving two or more parties:

... relations between Britain, France and Germany

I don't know where these "OA"s come from. There is no way that B can be correct on this item. "...
recounted several little-known episodes ... that illustrates ..." ?!

Here's what Webster's 10th Collegiate says about "between" vs. "among":

There is a persistent but unfounded notion that between can be used only with two items and that among
must be used with more than two. Between has been used of more than two since Old English: it is
especially appropriate to denote a one-to-one relationship, regardless of the number of items. It can be
used when the number is unspecified [economic cooperation between nations], when more than two are
enumerated [between you and me and the lamppost] [partitioned between Austria, Prussia, and Russia -Nathaniel Benchley] and even when only one item is mentioned (but repetition is implied) [pausing
between every sentence to rap the floor --George Eliot] Among is more appropriate when the emphasis is
on distribution rather than individual relationships [discontent among the peasants]. When among is
automatically chosen for more than two, English idiom may be strained [a worthy book that nevertheless
falls among many stools --John Simon] [the author alternates among mod slang, clichs and quotes from
literary giants --A.H. Johnson].

During her presidency of the short-lived Womans State Temperance Society (1852-1853), Elizabeth
Cady Stanton, as she was a staunch advocate of liberalized divorce laws, scandalized many of her
most ardent supporters in her suggestion that drunkenness should be made sufficient cause for
divorce.
A.as she was a staunch advocate of liberalized divorce laws, scandalized many of her most
ardent supporters in her suggestion that drunkenness should be
B. as she was a staunch advocate for liberalized divorce laws, scandalized many of her most
ardent supporters by her suggestion of drunkenness being
C. in being a staunch advocate for liberalized divorce laws, had scandalized many of her
most ardent supporters with the suggestion of drunkenness being

116

Adhamjons SC notes

UZBEKISTAN

D. a staunch advocate of liberalized divorce laws, scandalized many of her most ardent
supporters by suggesting that drunkenness be
E. a staunch advocate of liberalized divorce laws, she scandalized many of her most ardent
supporters in suggesting that drunkenness should be

EXPLANATION BY BOB:

Quote:
Originally Posted by balvinder
"drunkenness should be" or "drukenness be"

which is correct ...Please explain the correct way......

thanks

OA is D.

Subjunctive: "suggesting that drunkenness be." It is redundant to write "should" after

"suggesting."

Results of the recent study make it mandatory that the scientist rejects implementation of the
experimental procedure.
(A) rejects
(B) should reject
(C) reject
(D) must reject
(E) will reject
117

Adhamjons SC notes

UZBEKISTAN

EXPLANATION BY BOB:

It's the subjunctive. The present subjunctive is required in that clauses after verbs of requirement,
recommendation, and request. These verbs include: require, mandate, insist, command, demand, stipulate,
recommend, propose, suggest, urge, advise, move, request, ask, desire. The present subjunctive is also
used in that clauses after adjectives and nouns derived from these verbs: required, requirement, requested,
request, desirable, recommended, recommendation, proposed, proposal, suggested, suggestion, advised,
advisable, advice, motion, etc. It is also used in that clauses after other adjectives and nouns of
requirement, recommendation, and request: necessary, essential, critical, vital, imperative, urgent,
important, condition.

Here are some real GMAT Sentence Correction sentences that use the present subjunctive:

The Gorton-Dodd bill requires that a bank disclose to its customers how long it will delay access to funds
from deposited checks.

Legislation in the Canadian province of Ontario requires of both public and private employers that pay
be the same for jobs historically held by women as for jobs requiring comparable skill that are usually
held by men.

In one of the most stunning reversals in the history of marketing, the Coca-Cola company in July 1985
yielded to thousands of irate consumers who demanded that it bring back the original Coke formula.

The commission proposed that funding for the development of the park, which could be open to the public
early next year, be obtained through a local bond issue.

Most state constitutions now mandate that the state budget be balanced each year.

Despite protests from some waste-disposal companies, state health officials have ordered that the levels of
bacteria in seawater at popular beaches be measured and the results published.

Judicial rules in many states require that the identities of all prosecution witnesses be made known to
defendants so that they can attempt to rebut the testimony, but the Constitution explicitly requires only
118

Adhamjons SC notes

UZBEKISTAN

that the defendant have the opportunity to confront an accuser in court.

During her presidency of the short-lived Womans State Temperance Society (1852-1853), Elizabeth Cady
Stanton, a staunch advocate of liberalized divorce laws, scandalized many of her most ardent supporters
by suggesting that drunkenness be made sufficient cause for divorce.

Several financial officers of the company spoke on condition that they not be named in the press reports.

The new regulations mandate that a company allow its retiring employees who would otherwise lose
group health care coverage to continue the same insurance at their own expense for a specific period.

Bankers require that the financial information presented to them by mortgage applicants be complete and
follow a prescribed format

Emily Dickinsons letters to Susan Huntington Dickinson were written over a period beginning a
few years before Susans marriage to Emilys brother and ending shortly before Emilys death in
1886, outnumbering her letters to anyone else.
A. Dickinson were written over a period beginning a few years before Susans marriage to
Emilys brother and ending shortly before Emilys death in 1886, outnumbering
B. Dickinson were written over a period that begins a few years before Susans marriage to
Emilys brother and ended shortly before Emilys death in 1886, outnumber
C. Dickinson, written over a period beginning a few years before Susans marriage to Emilys
brother and that ends shortly before Emilys death in 1886and outnumbering
D. Dickinson, which were written over a period beginning a few years before Susans marriage
to Emilys brother, ending shortly before Emilys death in 1886, and outnumbering
E. Dickinson, which were written over a period beginning a few years before Susans marriage
to Emilys brother and ending shortly before Emilys death in 1886, outnumber
OA-E
Pls explain why
EXPLANATION BY BOB:

119

Adhamjons SC notes

UZBEKISTAN

"Which" refers unambiguously to "letters." "Which" cannot be used to refer to a person, and the following
verb "were" signals clearly that "which" is plural. Thus the only possible referent is "letters."

It is an oversimplification to say that "which" can be used to refer only to the word immediately preceding
the comma. Consider, for example, the following sentence from OG10 (SC #114):

From the bark of the paper birch tree the Menomini crafted a canoe about twenty feet long and two feet
wide, with small ribs and rails of cedar, which could carry four persons or eight hundred pounds of
baggage yet was so light that a person could easily portage it around impeding rapids.

Here the referent of "which" is is the quite distant "canoe."

Quote:
Originally Posted by Raj1972
Why A is wrong ?
Looks like E & A convey the same message and E is passive , A is active so whynot choose A

A is active? Really? "Emily Dickinson's letters ... were written ..." Can't be any more passive than that!

Who cares anyway? Passive is not wrong.

Hi,

Nuclear fusion is the force that powers the Sun, the stars, and hydrogen bombs, merging the nuclei
of atoms and not splitting them apart, as in nuclear reactors.

(A) merging the nuclei of atoms and not splitting them apart, as in nuclear reactors
(B) merging the nuclei of atoms instead of splitting them apart, like nuclear reactors
(C) merging the nuclei of atoms rather than splitting them apart, as nuclear reactors do
(D) and merges the nuclei of atoms but does not split them apart, as is done in nuclear reactors
(E) and merges the nuclei of atoms, unlike atomic reactors that split them apart.
120

Adhamjons SC notes

UZBEKISTAN

Official answer is E
why not D?

EXPLANATION BY BOB:

Quote:
Originally Posted by soz_gmat05
so what about C?

That's what OG 11th Edition says (p. 709).

Simplest way to rule out D and E is to say that you cannot have "and merges" after a comma. That's what
the OG explanation means when it says "incorrectly makes merges the second verb of the restrictive
clause... punctuation makes clear this separate action cannot be the case." If you want to say that nuclear
fusion is the force that does two things (powers and merges), you must not put a comma before "merges":

Nuclear fusion is the force that powers the Sun, the stars, and hydrogen bombs and merges...

D and E have other problems as well. D: "as is done in nuclear reactions" is not as good as "as nuclear
reactors do." E: "atomic reactors"? Where does this "atomic" come from?

What's wrong with C?

By a vote of 9 to 0, the Supreme Court awarded the Central Intelligence Agency broad discretionary
powers enabling it to withhold from the public the identities of its sources of intelligence
information.
(A) enabling it to withhold from the public
(B) for it to withhold from the public
(C) for withholding disclosure to the public of
121

Adhamjons SC notes

UZBEKISTAN

(D) that enable them to withhold from public disclosure


(E) that they can withhold public disclosure of

The OG answer is
SPOILER: A
.

I would like to know why the OG answer does not have a pronoun reference error. Does "it" refer back to
the Supreme Court or the CIA? Please respond with the rule to determine if the reference is sufficient.

EXPLANATION BY BOB:

Pronouns do not necessarily refer to the nearest eligible noun. If that were true, there would be no such
thing as ambiguous pronoun reference. The real reason that "it" is not ambiguous in choice A is that the
participle "enabling" refers to the Supreme Court, and so the object "it" of the participle refers to the
object of the Supreme Court's award, the CIA.

In the OG alone there are dozens of SC items in which the correct answer choice includes a pronoun
whose antecedent is not the nearest eligible noun. I have posted these before, but here again are seven
examples from OG10:

97. Iguanas have been an important food source in Latin America since prehistoric times, and they are
still prized as game animals by the campesinos....

The pronoun they refers to iguanas, not to the nearest eligible noun times.

103. Students in the metropolitan school district are so lacking in math skills that it will be difficult to
absorb them into a city economy....

The pronoun them refers to students, not to the nearest eligible noun skills.

122

Adhamjons SC notes

UZBEKISTAN

152. ...a microbe never before seen on Earth that might escape from the laboratory and kill vast numbers
of humans who would have no natural defenses against it.

The pronoun it refers to microbe, not to the nearest eligible noun laboratory.

153. A recording system was so secretly installed and operated in the Kennedy Oval Office that even
Theodore C. Sorensen, the White House counsel, did not know it existed.

The pronoun it refers to system, not to the nearest eligible noun Office.

159. While depressed property values can hurt some large investors, they are potentially devestating for
homeowners...

The pronoun they refers to values, not to the nearest eligible noun investors.

180. Quasars are so distant that their light has taken billions of years to reach the Earth; consequently,
we see them as they were during the formation of the universe.

The pronouns them and they refer to quasars, not to the nearest eligible noun years.

251. The gyrfalcon, an Arctic bird of prey, has survived a close brush with extinction; its numbers are now
five times greater than when....

The pronoun its refers to gyrfalcon, not to the nearest eligible noun extinction.

Quote:
Originally Posted by mtr
Answer (B) doesn't look so bad to me. What is the error?

123

Adhamjons SC notes

UZBEKISTAN

"...powers for it to withhold..." is unidiomatic.


Quote:
Originally Posted by kaustavmishra
why are options D & E wrong

D and E are wrong because the plural pronouns "them" and "they" do not agree with the singular
antecedent "Central Intelligence Agency."

Quote:
Originally Posted by JASH
If I understand your explanation correctly: an object pronoun (e.g. him, her, them, it, whom) will always
refer to the object and as long as the reference to the object is unambiguous (clearly reference one of
multiple objects if there are multiple objects) we are fine.

Actually, what I was trying to say is that if a pronoun is an object of a verb, then the pronoun cannot refer
to the subject of that verb.

Quote:
Originally Posted by DeusExMachina
Hi 800Bob,

Shouldn't the subject of the participle "enabling" be the "broad discretionary powers"? I thought that the
powers enable the CIA to withold information from the public, rather than the Supreme Court that
awarded them. Thus, the pronoun "it" could still refer to the earlier subject of "the Supreme Court"?

I will agree with you. The subject of "enabling" is really "powers." In the end, I think what we have here is
an example of a situation in which those who are obsessed with finding ambiguous pronoun reference will
say that "it" is ambiguous. The fact is, ambiguous reference is one of those grey-area issues. The rules of
usage and style are not neat and tidy like mathematical rules. Sometimes common sense enters into the
picture. In this case, how could anyone think that the Supreme Court could award powers to the CIA that
enable the Supreme Court to withhold identities? And if one were to insist that the pronoun is ambiguous,
what other choice would one go for?
124

Adhamjons SC notes

UZBEKISTAN

International sporting events need not be fiscal disasters, the financial success of the 1992 Olympic
Games demonstrates that fact.

(A) the financial success of the 1992 Olympic Games demonstrates that fact
(B) for example, the 1992 Olympic Games were financially successful
(C) like the financial success of the 1992 Olympic Games demonstrates
(D) a fact demonstrated by the 1992 Olympic Games, which were financially successful
(E) as the financial success of the 1992 Olympic Games demonstrates.

Points to note: 'demonstrate' is a transitive verb AND 1992 Olympic Games are singular or plural?

EXPLANATION BY BOB:

Quote:
Originally Posted by shud
The issue of discussion is arround the elimination of option B and D where were is used for olympic
games.

B and D are wrong, but not because of subject-verb agreement.

B is absolutely impossible because it is a run-on sentence. For example is not a conjunction.


D is grammatically correct, but it is unnecessarily wordy and not so logically constructed as E. Its not
really the 1992 Games that demonstrate the fact; its their financial success that demonstrates the fact. In
D the phrase which were financially successful seems tacked on like a piece of extra information.

Question 120 in OG10 is an easy question, but reading through the answers to 120. The explanation for E
(which is the wrong answer) is hard to understand; the explanations states: "and in E, 'establishing and
using modify Athens,"

125

Adhamjons SC notes

UZBEKISTAN

Here's the complete sentence if it were written with E as the answer:

"The Parthenon was a church from 1204 until 1456, when Athens was taken by General Mohammed the
Conqueror, the Turkish sultan, establishing a mosque in the building and using the Aeropolis as a
fortress."

Why is "Athens" the referent to the partipial, "establishing a..."?

Take two other examples in OG, namely 179 and 191.

Again, these are not the correct choices, but it's the seemingly contradictory explanations between 120 and
179/191 that is confusing.

179)
Choice D) During the early years of European settlement on a continent that was viewed as "wilderness"
by the newcomer, having intimate knowledge of the ecology of the land, Native Americans helped the
rescue of many Pilgrams and pioneers from hardship, or even death.

Choice E) During the early years of European settlement on a continent that was viewed as "wilderness"
by the newcomer, knowing intimately the ecology of the land, Native Americans helped to rescue many
Pilgrams and pioneers from hardship, or even death.

179 Explanation for D and E: "D and E are confusingly worded because they begin with present
participles (having and knowing) that appear at first to refer to the immediately preceding noun,
newcomers, rather than to Native Americans.

In this expample the participle refers to the immediately preceding noun. Here's another explanation that
does as well.

191 Choice D, the wrong answer, but the explanation and how it differs with 120 is where I would
welcome insight.

191 with D inserted) "Lawmakers are examining measures that would require banks to disclose all fees
and account requirements in writing, to provide free cashing of government checks, creating basic savings
126

Adhamjons SC notes

UZBEKISTAN

accounts to carry minimal fees and require minimal initial deposits."

Explanation for D): "a modification problem results because the participial phrase creating... ataches to
the noun checks, thus distorting the..."

Here again the participial refers to the noun immediately before it. I can give you many more examples
where the referent to the participial phrase refers to the noun immediately preceding. The exceptions are
few and make complete sense to me with the exception of 120. Can someone explain what makes 120
different, and more importantly how to identify the referent?

EXPLANATION BY BOB:

On the basis of gschmilinsky's observations, I am prepared to revise my attempt at a general rule. I now
say:

"A participial phrase located at the beginning or the end of a clause and set off from that clause with
commas generally modifies the subject of the clause."

Thus 120(E) "...when Athens was taken by General Mohammed the Conqueror, the Turkish sultan,
establishing a Mosque in the building and using the Acropolis as a fortress" is problematic because the
grammar rule says that the phrase refers to "Athens," but common sense says it refers to "sultan."

The explanation to 179 says of D and E that the participial phrases "appear at first to refer to the
immediately preceding noun." As one reads further, one realizes that the participial phrase in each case is
at the beginning of a clause, and so it later appears to modify the following subject. That's what is called a
"squinting modifier" -- one that is located between two possible referents and requiring the reader to work
a little extra hard to figure out what the writer intended.

As for 191 (D), I'm not sure what the explanation means by "the participial phrase creating... attaches to
the nouns checks." I think the writer of the explanation is intentionally avoiding the word "modifies" here.
Choice (D) is impossible in any case because of the missing "and." If the sentence had been:

"Lawmakers are examining measures that would require banks to disclose all fees and account
127

Adhamjons SC notes

UZBEKISTAN

requirements in writing and to provide free cashing of government checks, creating basic service accounts
to carry minimal fees and require minimal initial deposits."

...then I would say that the participial phrase creating... modifies (illogically) the subject that (=
measures).

In any case, I think gschmilinsky has shown that grammar rules are not so clearcut as math rules. I can
find no reputable grammar source that provides a scientific explanation for identifying the antecedent of a
pronoun or the referent of a modifier. All I find is vague rules such as "make a pronoun refer clearly to
one antecedent" and "place modifiers where they will clearly modify the words intended."

1. A collection of 38 poems by Phillis Wheatley, a slave, was published in the 1770s, the first book
by a Black woman and it was only the second published by an American woman.
(A) it was only the second published by an American woman
(B) it was only the second that an American woman published
(C) the second one only published by an American woman
(D) the second one only that an American woman published
(E) only the second published by an American woman
2. A firm that specializes in the analysis of handwriting claims from a one-page writing sample that
it can assess more than three hundred personality traits, including enthusiasm, imagination, and
ambition.
(A) from a one-page writing sample that it can assess
(B) from a one-page writing sample it has the ability of assessing
(C) the ability, from a one-page writing sample, of assessing
(D) to be able, from a one-page writing sample, to assess
(E) being able to assess, from a one-page writing sample,
3. A huge flying reptile that died out with the dinosaurs some 65 million years ago, the
Quetzalcoatlus had a wingspan of 36 feet, believed to be the largest flying creature the world has
ever seen.
(A) believed to be
128

Adhamjons SC notes

UZBEKISTAN

(B) and that is believed to be


(C) and it is believed to have been
(D) which was, it is believed,
(E) which is believed to be
4. A recent national study of the public schools shows that there are now one microcomputer for
every thirty-two pupils, four times as many than there were four years ago.
(A) there are now one microcomputer for every thirty-two pupils, four times as many than there
were
(B) there is now one microcomputer for every thirty-two pupils, four times as many than there were
(C) there is now one microcomputer for every thirty-two pupils, four times as many as there were
(D) every thirty-two pupils now have one microcomputer, four times as many than there were
(E) every thirty-two pupils now has one microcomputer, four times as many as
4. A recent study has found that within the past few years, many doctors had elected early
retirement rather than face the threats of lawsuits and the rising costs of malpractice insurance.
(A) had elected early retirement rather than face
(B) had elected early retirement instead of facing
(C) have elected retiring early instead of facing
(D) have elected to retire early rather than facing
(E) have elected to retire early rather than face
5. A recent study of ancient clay deposits has provided new evidence supporting the theory of global
forest fires ignited by a meteorite impact that contributed to the extinction of the dinosaurs and
many other creatures some 65 million years ago.
(A) supporting the theory of global forest fires ignited by a meteorite impact that
(B) supporting the theory that global forest fires ignited by a meteorite impact
(C) that supports the theory of global forest fires that were ignited by a meteorite impact and that
(D) in support of the theory that global forest fires were ignited by a meteorite impact and that
(E) of support for the theory of a meteorite impact that ignited global forest fires and

Please explain ur replies...thanx!!!

OA-E,D,CC,E,B

EXPLANATION BY BOB:
129

Adhamjons SC notes

UZBEKISTAN

Quote:
Originally Posted by shud
iknownoone,

Thats exactly why I posted my question.


The question, I posted is from OG 10 ( question 228) and correct answer is B -'are believed to be'.
My worry is question posted by hitz has similar construction but there isn't any option having 'is believed
to be', so got confused.

May be, for hitz question B- 'and that is believed to be' is correct answer because of llelism of 'that...and
that is ' . What's the OA hitz, I am curious.

Can anybody tell me whats the difference between the two Qs.

The difference is that the Quetzalcoatlus is extinct while quasars still exist.

The Quetzalcoatlus is believed to have been... is correct because the believing is happening in the present
and the Quetzalcoatlus existed in the past.

Quasars are believed to be... is correct because the believing is happening in the present and quasars exist
in the present.

Until his battle fleet and transports were destroyed by the Royal Navy, Napoleon had been
intending the invasion of Britain.

A had been intending the invasion of


B was intending the invasion of
C had intended invading
D had intend to invade
E intended invading
130

Adhamjons SC notes

UZBEKISTAN

EXPLANATION BY BOB:

I see no good answer. (A) and (B) are unidiomatic because "intend" in the sense of "plan" is not used
transitively; I have never heard or read anything like "intend the invasion of." (C) and (E) are unidiomatic
because of the "-ing" form; "intend" takes the infinitive: the correct idiom is "intend to do something." (D)
is ungrammatical because after "had" we need "intended." I would say: "had intended to invade."

I have done some research and found several sources that say that "intend" can take either the infinitive or
the gerund, with no change in meaning. To my ear "intended invading" sounds substandard, but I guess
enough people use that form now that it has become accepted. It doesn't sound like the GMAT to me, but
in any case, I now concur that the best of the choices given is C.

The automobile company announced that the average price of next years models would decrease fourtenths of one percent, amounting to about $72, as compared with comparably equipped cars and trucks
this year.

A. The automobile company announced that the average price of next years models would decrease fourtenths of one percent, amounting to about $72, as compared with comparably equipped cars and trucks
this year.
B. The automobile company announced that the average price of next years cars and trucks would
decrease four-tenths of one percent, or about $72, from that of comparably equipped models this year.
C. The automobile company announced that there would be a decrease of four-tenths of one percent in
next years models price, or about $72, below this years comparably equipped cars and trucks.
D. Compared to comparably equipped models this year, the automobile company announced that the
average price of next years cars and trucks would decrease about $72, less by four-tenths of one percent.
E. Contrasted with comparably equipped cars and trucks of this year, the automobile company announced
that the average price of next years models would decrease about $72, or four-tenths of one percent.

131

Adhamjons SC notes

UZBEKISTAN

A recent review of pay scales indicates that CEOs now earn an average of 419 times more pay than bluecollar workers, compared to a ratio of 42 times in 1980.

A. that CEOs now earn an average of 419 times more pay than blue-collar workers, compared to a
ratio of 42 times
B. that, on average, CEOs now earn 419 times the pay of blue-collar workers, a ratio that compares
to 42 times
C. that, on average, CEOs now earn 419 times the pay of blue-collar workers, as compared to 42
times their pay, the ratio
D. CEOs who now earn on average 419 times more pay than blue-collar workers, as compared to 42
times their pay, the ratio
E. CEOs now earning an average of 419 times the pay of blue-collar workers, compared to the ratio
of 42 times

Question 1 : B is good.
D and E - modifer problem
A is awkward
C - there would be - never use this in GMAT
Hence B.

2-QUESTION:

BOBS EXPLANATION:

I wish to change my answer to C.

(B) ...CEO's now earn 419 times the pay of blue-collar workers, a ratio that compares to 42 times in 1980.

The problem with this choice is "42 times in 1980," which is confusing because it sounds at first as if
something happened 42 times in 1980. It is also an incomplete comparison: "419 times the pay of bluecollar workers" versus "42 times."
132

Adhamjons SC notes

UZBEKISTAN

(C) ...CEO's now earn 419 times the pay of blue-collar workers, as compared to 42 times their pay, the
ratio in 1980.

I stated above that the reference of "their" is ambiguous, but only a nitpicker would have any problem
understanding that "their pay" is parallel to "the pay of blue-collar workers."

This item is instructive in two regards. First, it shows once again that ambiguous pronoun reference is just
not as big an issue for GMAT writers as it is for us Test Magic types. Second, it shows that GMAT writers
do not subscribe to the rules I was taught about the difference between "compared with" and "compared
to."

OTHER GUY:

"It is also an incomplete comparison: "419 times the pay of blue-collar workers" versus "42 times."

I believe that this point of 800Bob's regarding maintaining parallelism is the more important one and the
easier one to identify during an exam.

6 SCs from GMATPrep. Hope be helpful !

1.Many environmentalists, and some economists, say that free trade encourages industry to relocate to
countries with ineffective or poorly enforced antipollution laws, mostly in the developing world, and that,
in order to maintain competitiveness, rich nations have joined this downward slide toward more lax
attitudes about pollution.

A. that, in order to maintain competitiveness, rich nations have joined this


downward slide toward more lax attitudes about pollution
B. that, for maintaining competitiveness, rich nations join in this downward slide
toward more lax attitudes about pollution
133

Adhamjons SC notes

UZBEKISTAN

C. that rich nations join this downward slide toward more lax attitudes about
pollution because of wanting to maintain competitiveness
D. that in rich nations, joining this downward slide toward more lax attitudes
about pollution is a result of wanting maintain competition
E. that wanting to maintain competition is making rich nations join in this
downward slide toward an attitude about pollution that is more lax

2.The largest trade-book publisher in the United States has announce the creation of a new digital imprint
division,under which it will publish about 20 purely digital works to be sold online as either electronic
books or downloadable copies that can be printed upon purchase.

A.works to be sold online as either electronic books or


B.works to sell them online, either as electronic books or
C.works and it will sell them online as either electronic books or as
D.works, and selling them online as either electronic books or as
E.works, and it will sell them online either as electronic books or

3.The investigations of many psychologists and anthropologists support the generalization of there being
little that is a significant difference in the underlying mental processes manifested by people from
different culture.

A. of there being little that is a significant difference


B. of there being little that is significant different
C. of little that is significantly different
D. that there is little that is significantly different
E. that there is little of significant differences

4.Paleontologist Stephen Jay Gould has argued that many biological traits are not the products of natural
selection, favored due to their enhancement of reproduction or survival, but that they are simply random
by-products of other evolutionary developments.

134

Adhamjons SC notes

UZBEKISTAN

A.due to their enhancement of reproduction or survival, but that they are


B.due to the reproduction or survival they enhance, but they are
C.because they enhance reproduction or survival, but
D.because they enhance reproduction or survival, but are
E.because of enhancing reproduction or survival, but are

5.One report concludes that many schools do not have, or likely to have, enough computers to use them
effectively.
A.or
B.nor
C.or are
D.nor are they
E.nor are not

6.The Nobel Prize in chemistry was awarded to three scientists for their discovery that plastic can be made
electrically conductive- an advance that has led to improvements in film, television screen, and windows.

A. that plastic can be made electrically conductive- an advance that has led
B.that plastic can be made electrically conductive- this advance leading
C.that plastic can be made to be electrically conductive- and this advance led
D.of plastic's ability to be made electrically conductive, with this advance
leading
E.of plastic being able to be made electrically conductive- an advance that has
led

OA from GMATPrep - 1.A 2.A 3.D 4.D 5.D 6.A.

kunalguhaS EXPLANATIONS:

Hi I have marked what I thought was incorrect in red.

1.Many environmentalists, and some economists, say that free trade encourages industry to relocate to
countries with ineffective or poorly enforced antipollution laws, mostly in the developing world, and that,
in order to maintain competitiveness, rich nations have joined this downward slide toward more lax
135

Adhamjons SC notes

UZBEKISTAN

attitudes about pollution.

A. that, in order to maintain competitiveness, rich nations have joined this


downward slide toward more lax attitudes about pollution (this answer choice would be the best)
B. that, for maintaining competitiveness, rich nations join in this downward slide for maintaining doesnt
seem correct
toward more lax attitudes about pollution
C. that rich nations join this downward slide toward more lax attitudes about
pollution because of wanting to maintain competitiveness
D. that in rich nations, joining this downward slide toward more lax attitudes
about pollution is a result of wanting maintain competition
E. that wanting to maintain competition is making rich nations join in this
downward slide toward an attitude about pollution that is more lax

2.The largest trade-book publisher in the United States has announce the creation of a new digital imprint
division,under which it will publish about 20 purely digital works to be sold online as either electronic
books or downloadable copies that can be printed upon purchase.

A.works to be sold online as either electronic books or


B.works to sell them online, either as electronic books or
C.works and it will sell them online as either electronic books or as (is the
D.works, and selling them online as either electronic books or as
E.works, and it will sell them online either as electronic books or

C and E change the meaning of the sentence by saying the book publisher would be physically selling
books.

3.The investigations of many psychologists and anthropologists support the generalization of there being
little that is a significant difference in the underlying mental processes manifested by people from
different culture.

A. of there being little that is a significant difference


B. of there being little that is significant different(being is wrong)
136

Adhamjons SC notes

UZBEKISTAN

C. of little that is significantly different


D. that there is little that is significantly different
E. that there is little of significant differences ( little of differences is awkward)
Support the generalization that would be correct.

4.Paleontologist Stephen Jay Gould has argued that many biological traits are not the products of natural
selection, favored due to their enhancement of reproduction or survival, but that they are simply random
by-products of other evolutionary developments.

A.due to their enhancement of reproduction or survival, but that they are


B.due to the reproduction or survival they enhance, but they are (due to should only be used to mean
caused by )
C.because they enhance reproduction or survival, but
D.because they enhance reproduction or survival, but are (this is the best)
E.because of enhancing reproduction or survival, but are (E is incorrect as it says that the traits are favored
by enhancing reproduction. i.e. enhancing reproduction is used as a noun that favours biological traits)
The beacuse of seems awkward

Biological traits are not the products .but are..

They are favoured because they enhance reproduction would be correct.

5.One report concludes that many schools do not have, or likely to have, enough computers to use them
effectively.
A.or
B.nor
C.or are
D.nor are they (Use of the not before requires a nor) the they clearly refers to schools in the comparision)
This one feels right
E.nor are not

6.The Nobel Prize in chemistry was awarded to three scientists for their discovery that plastic can be made
electrically conductive- an advance that has led to improvements in film, television screen, and windows.
137

Adhamjons SC notes

UZBEKISTAN

A. that plastic can be made electrically conductive- an advance that has led (this is the most concise)
B.that plastic can be made electrically conductive- this advance leading
C.that plastic can be made to be electrically conductive- and this advance led
D.of plastic's ability to be made electrically conductive, with this advance
leading
E.of plastic being able to be made electrically conductive- an advance that has
led
^^^^^^^^^^^^^^^^^^^^^^^^^^^^^^^^^^^^^^^^^^^^^^^^^^ ^^^^^^^^^

Discovery that would be correct here.hence A is the best ans.

As a result of the ground-breaking work of Barbara McClintock, many scientists now believe that
all of the information encoded in 50.000 to 100.000 of the different genes found in a human cell are
contained in merely three percent of the cell's DNA.
1. 50,000 to 100,000 of the different genes found in a human cell are contained in merely
2. 50,000 to 100,000 of the human cell's different genes are contained in a mere
3. the 50,000 to 100,000 different genes found in human cells are contained in merely
4. 50,000 to 100,000 of human cells' different genes is contained in merely
5. the 50,000 to 100,000 different genes found in a human cell is contained in a mere

EXPLANATION BY BOB:

The best response is E.

D is wrong, but not really because of the possessive construction used with an inanimate object. Here are a
few very common examples of such a construction:

New Year's Day


a month's salary
the water's edge
138

Adhamjons SC notes

UZBEKISTAN

today's newspaper

In this particular SC the possessive form is awkward, but that alone is not enough to eliminate D. The
clincher is that D distorts the intended meaning by changing the singular "cell" into the plural "cells." The
sentence speaks of the 50,000 to 100,000 genes in one cell. To be accurate D would have to read: "50,000
to 100,000 of a human cell's different genes is contained in merely..."

D is problematic. Nothing wrong with E. So the best response is E.

In the Louisiana Purchase of 1803, the United States acquired 828,000 square miles for about four
cents an acre, which more than doubled the country's size and that brought its western border
within reach of the Pacific Ocean.
A: In the Louisiana Purchase of 1803, the United States acquired 828,000 square miles for about
four cents an acre, which more than doubled the country's size and that brought
B: For about four cents an acre the United States acquired, in the Louisiana Purchase of 1803,
828,000 square miles, more than doubling the country's size and it brought
C: With the Louisiana Purchase in 1803, the United States acquired 828,000 square miles for about
four cents an acre, more than doubling its size and bringing
D: The United States, in the Louisiana Purchase of 1803, for about four cents an acre, acquired
828,000 square miles, more than doubling the country's size, bringing
E: Acquiring 828,000 square miles in the Louisiana Purchase of 1803, the United States bought it
for about four cents an acre, more than doubling the country's size and bringing

EXPLANATION BY BOB:

Quote:
Originally Posted by yoda_ngen
In C, we are using possessive pronoun its which does not seem to replace any possessive noun.

139

Adhamjons SC notes

UZBEKISTAN

I don't understand your point. If "its" is wrong in choice C, how do you explain the "its" that follows in the
non-underlined portion of the sentence. It's pretty clear to me that "its" refers to "United States."

Quote:
Originally Posted by kamal_gmat2005
bob what would be ur pick....

The correct response is C.

According to entomologists, single locusts are quiet creatures, but when locusts are placed with
others of their species, they become excited, change color, vibrate, and even hum.

A.when locusts are placed with others of their species,


B.when placing them with others of their species, then
C.locusts, when placed with others of their species,
D.placing them with others of their species,
E.placed with others of their species, then

I dont have OA for this.

Reference answer is
SPOILER: A

EXPLANATION BY BOB:

Quote:
Originally Posted by ramyagupta
U can sneak a clause right between the conjunction (BUT) and the rest of the sentence.

140

Adhamjons SC notes

UZBEKISTAN

Yes, you can insert a clause between the conjunction and the rest of the sentence, but C is wrong. Look at
what you have with C if you omit the inserted clause: "...but locusts they become excited." That's a double
subject.

There's nothing wrong with A. "They" refers quite clearly to "locusts," the subject of the preceding clause
and the antecendent of the possessive "their."

Quote:
Originally Posted by ramyagupta
I totally agree with you.
But in the answer A dont you think there is a comma missing ?

but, when locusts are placed with others of their species,

In modern usage, the comma is usually omitted in such situations. From


http://laurentgrenier.com/grammar&style.html:

Dont ordinarily put a comma after a conjunction just because what follows is an introductory
word, phrase, or clause. This rule applies not only to coordinating conjunctions such as and, but,
and or but to subordinating conjunctions such as that, if, and when.

The wind had risen, and, throughout the night, the rain beat against the windows and The storm was
over, but, in its aftermath, the heavy rain continued are overpunctuated. The and in the first sentence
and the but in the second are coordinating conjunctions, connecting independent clauses. There is
no reason to have commas after them; a conjunction should not have its joining function
contradicted by a comma.

General principle of modern punctuation: to use punctuation lightly and omit it when the signal it
would give is false or unnecessary.

He said that, although he couldnt stay long, he would come. There is no justification for the comma
after that. The entire word group although he couldnt stay long, he would come is the object of He
said, and putting a comma in front of it is like improperly separating verb and object, though the
141

Adhamjons SC notes

UZBEKISTAN

intervening that makes it less obvious that the separation is being committed. The comma after long,
marking the end of the introductory clause of the two-clause word group, is optional but quite
desirable, both because the clause is not very short and because the meanings of the clauses it
separates are opposed.

I should add: I have never seen a GMAT Sentence Correction answer choice that was wrong only because
of a missing comma. Punctuation is not a major issue in these questions.

Many of them chiseled from solid rock centuries ago, the mountainous regions of northern Ethiopia
are dotted with hundreds of monasteries.

(A) Many of them chiseled from solid rock centuries ago, the mountainous regions of northern
Ethiopia are dotted with hundreds of monasteries.

(B) Chiseled from solid rock centuries ago, the mountainous regions of northern Ethiopia are dotted
with many hundreds of monasteries.

(C) Hundreds of monasteries, many of them chiseled from solid rock centuries ago, are dotting the
mountainous regions of northern Ethiopia.

(D) The mountainous regions of northern Ethiopia are dotted with hundreds of monasteries, many
of which are chiseled from solid rock centuries ago.

(E) The mountainous regions of northern Ethiopia are dotted with hundreds of monasteries, many
of them chiseled from solid rock centuries ago.

EXPLANATION BY BOB:

The questions in 1000SC are not all real questions, and even the ones that are sometimes contain typos.
There is no good answer here. D is wrong because "are" should be "were." E is wrong because "them" is
ambiguous.

142

Adhamjons SC notes

UZBEKISTAN

Please Explain 10 real SC

1. That twenty-one ceramic dog figurines were discovered during the excavating of a 1,000-year-old
Hohokam village in Tempe, Arizona, has nearly doubled the number of these artifacts known to
exist.

A. That twenty-one ceramic dog figurines were discovered during the excavating
B. Twenty-one ceramic dog figurines discovered at the excavation
C. Discovering twenty-one ceramic dog figurines at the excavating
D. Ceramic dog figurines, twenty-one of which were discovered during excavating
E The discovery of twenty-one ceramic dog figurines during the excavation

2. Faulty voting equipment, confusing ballots, voter error, and problems at polling places have been
cited by a new study of the 2000 United States presidential election, which estimated that they did
not count 4 million to 6 million of the 100 million votes cast.

A. Faulty voting equipment, confusing ballots, voter error, and problems at polling places have been
cited by a new study of the 2000 United States presidential election, which estimated that they did
not count 4 million to 6 million of the 100 million votes cast.
B. Citing faulty voting equipment, confusing ballots, voter error, and problems at polling places, a
new study of the 2000 United States presidential election has estimated that 4 million to 6 million of
the 100 million votes cast were not counted.
C. Citing faulty voting equipment, confusing ballots, voter error, and problems at polling places, 4
million to 6 million of the 100 million votes cast were not counted in the 2000 United States
presidential election, a new study estimates.
D. A new study has cited faulty voting equipment, confusing ballots, voter error, and problems at
polling places in estimating that 4 million to 6 million of the 100 million votes that were cast had not
been counted in the 2000 United States presidential election.
E. A new study of the 2000 United States presidential election, citing faulty voting equipment,
confusing ballots, voter error, and problems at polling places, has estimated 4 million to 6 million
votes had not been counted of the 100 million votes cast.
143

Adhamjons SC notes

UZBEKISTAN

3. To develop more accurate population forecasts, demographers have to know a great deal more
than now about the social and economic determinants of fertility.

A. have to know a great deal more than now about the social and economic
B. have to know a great deal more than they do now about the social and economical
C. would have to know a great deal more than they do now about the social and economical
D. would have to know a great deal more than they do now about the social and economic
E. would have to know a great deal more than now about the social and economical

4. Although people in France consume fatty foods at a rate comparable to the United States, their
death rates from heart disease are far lower in France.

A. people in France consume fatty foods at a rate comparable to the United States, their
B. people in France and the United States consume fatty foods at about the same rate, the
C. fatty foods are consumed by people in France at a comparable rate to the United Statess, their
D. the rate of fatty foods consumed in France and the United States is about the same, the
E. the rate of people consuming fatty foods is about the same in France and the United States, the

5. Only seven people this century have been killed by the great white shark, the man-eater of the
moviesless than those killed by bee stings.

A. moviesless than those


B. moviesfewer than have been
C. movies, which is less than those
D. movies, a number lower than the people
E. movies, fewer than the ones

6. Since February, the Federal Reserve has raised its short-term interest rate target five times, and
because of the economys continued strength, analysts have been predicting for weeks that the
target will be raised again in November.

A. because of the economys continued strength, analysts have been predicting for weeks that the
target will
144

Adhamjons SC notes

UZBEKISTAN

B. with the economys strength continuing, analysts predicted for weeks that the target
C. because the economy continues strong, analysts predicted for weeks that the target would
D. due to the economys continued strength, analysts have been predicting for weeks that the target
due to the fact of the economys continued strength, analysts predicted for weeks that the target will

7. A surge in new home sales and a drop in weekly unemployment claims suggest that the economy
might not be as weak as some analysts previously thought.

A. claims suggest that the economy might not be as weak as some analysts previously thought
B. claims suggests that the economy might not be so weak as some analysts have previously thought
C. claims suggest that the economy might not be as weak as have been previously thought by some
analysts
D. claims, suggesting about the economy that it might not be so weak as previously thought by some
analysts
E. claims, suggesting the economy might not be as weak as previously thought to be by some
analysts
Answer:
------------------------------------------------------------------------------------------------------------

8. The health benefits of tea have been the subject of much research; in addition to its possibilities
for preventing and inhibiting some forms of cancer, the brewed leaves of Camellia sinensis may also
play a role in reducing the risk of heart disease and stroke.

A. in addition to its possibilities for preventing and inhibiting


B. in addition to its possibilities to prevent or inhibit
C. besides the possibility that it prevents and inhibits
D. besides the possible preventing and inhibiting of
E. besides possibly preventing or inhibiting

9. A leading figure in the Scottish enlightenment, Adam Smiths two major books are to democratic
capitalism what Marxs Das Kapital is to socialism.

A. Adam Smiths two major books are to democratic capitalism what


145

Adhamjons SC notes

UZBEKISTAN

B. Adam Smiths two major books are to democratic capitalism like


C. Adam Smiths two major books are to democratic capitalism just as
D. Adam Smith wrote two major books that are to democratic capitalism similar to
E. Adam Smith wrote two major books that are to democratic capitalism what

10. The first trenches that were cut into a 500-acre site at Tell Hamoukar, Syria, have yielded strong
evidence for centrally administered complex societies in northern regions of the Middle East that
were arising simultaneously with but independently of the more celebrated city-states of southern
Mesopotamia, in what is now southern Iraq.

A. that were cut into a 500-acre site at Tell Hamoukar, Syria, have yielded strong evidence for
centrally administered complex societies in northern regions of the Middle East that were arising
simultaneously with but
B. that were cut into a 500-acre site at Tell Hamoukar, Syria, yields strong evidence that centrally
administered complex societies in northern regions of the Middle East were arising simultaneously
with but also
C. having been cut into a 500-acre site at Tell Hamoukar, Syria, have yielded strong evidence that
centrally administered complex societies in northern regions of the Middle East were arising
simultaneously but
D. cut into a 500-acre site at Tell Hamoukar, Syria, yields strong evidence of centrally administered
complex societies in northern regions of the Middle East arising simultaneously but also
E. cut into a 500-acre site at Tell Hamoukar, Syria, have yielded strong evidence that centrally
administered complex societies in northern regions of the Middle East arose simultaneously with
but

EXPLANATION BY BOB:

1. E

2. B

3. D
146

Adhamjons SC notes

UZBEKISTAN

4. B

5. B I rule out D because "number" cannot be compared with "people." I rule out E because "fewer than
the ones" is unidiomatic.

6. A One should avoid using "due to" as a synonym of "because of."

7. A Choice C is wrong because "have been previously thought by some analysts" is an ugly and
unnecessary passive construction, and because "have" should be singular "has."

8. E

9. E Because of the introductory modifying phrase, the subject must be "Adam Smith," not "books."

10. E

4. D is wrong because "rate of fatty foods consumed" is imprecise. Foods can't have a rate. The rate is not
of foods but of consumption. And there are two rates. Should be something more like "the rates at which
fatty foods are consumed in France and the United States are about the same..."

5. "People" is countable. Example: "There are five people in the room."

7. It's a double subject: "A surge... and a drop... suggest..."

I think that, for question 5, C is wrong because "which" would refer illogically to the immediately
preceding noun "shark." In B the verb "have been" seems to make it clear that "fewer" refers to the subject
"people." "Seven people have been killed by sharks--fewer than have been killed by bees.

Q: By the way,about question number 6; some guys say D but I took A!


Could you explain why we cannot use D?

147

Adhamjons SC notes

UZBEKISTAN

"Due to economy's continued strength ..." = "caused by economy's continued strength ..."

ANSWER:
Because the analysts are not "due to" (caused by) the economy's continued strength.

Historians have long debated whether the spectacular rise and fall of Napoleon in the decades after
the French Revolution was the necessary outcome of the political vacuum having been created by
the toppled monarchy.

A) was the necessary outcome of the political vacuum having been created by the toppled monarchy

B) was the necessary outcome of the political vacuum created by toppling the monarchy

C) were the necessary outcome of the political vacuum that the toppled monarchy created

D) was the necessary outcome of the political vacuum created from toppling the monarchy

E) were the necessary outcome of the political vacuum created by the toppling of the monarchy

EXPLANATION BY BOB:

Rise and fall" seems to function as a singular subject here, as in: "Bacon and eggs is my favorite
breakfast."
One thing that makes "rise and fall" feel singular to me is the use of just one determiner. I think of "the
rise and fall" as singular, and "the rise and the fall" as plural. Another reason "rise and fall" feels singular
to me is that the complement is singular. If "rise" and "fall" were really two separate subjects, I would
prefer to say that they were "outcomes."

The rise and fall was the outcome.


The rise and the fall were outcomes.
148

Adhamjons SC notes

UZBEKISTAN

In any case, I wouldn't worry too much item since it is clearly not of GMAT quality. (Smells like Kaplan
to me.)

The Achaemenid empire of Persia reached the Indus Valley in the fifth century B.C., bringing the
Aramaic script with it, from which was derived both northern and southern Indian alphabets.

A the Aramaic script with it, from which was derived both northern and

B the Aramaic script with it, and from which deriving both the northern and the

C with it the Aramaic script, from which derive both the northern and the

D with it the Aramaic script, from which derives both northern and

E with it the Aramaic script, and deriving from it both the northern and

EXPLANATION BY BOB:

The correct response is C.

A and B can be eliminated because "which" refers to "it" (Achaemenid empire) instead of "Aramaic
script."

D is wrong because the singular verb "derives" does not agree with the plural subject "alphabets."

E is wrong because "deriving" seems to refer to the subject of the sentence "Achaemenid empire."

Here are a couple, one a 500-600 lvl and another a 700-800 level (as marked by Manhattan). I got them
wrong and am still unclear about why my answer choices were wrong. Hopefully some of you can better
149

Adhamjons SC notes

UZBEKISTAN

explain them, so here goes:

PROBLEM 1:

Since 1989, after the Berlin Wall had been demolished, one of the most problematic ethnic groups in
the reunified Germany, in cultural and economic assimilation terms, were the former East
Germans, who have had to acclimate to an entirely different political system.

A)after the Berlin Wall had been demolished, one of the most problematic ethnic groups in the
reunified German, in cultural and economic assimilation terms, were the former East Germans

B)after the Berlin Wall was demolished, one of the most problematic ethnic groups in the reunified
Germany, in cultural terms as well as those of economic assimilation, were the former East
Germans

C)when the Berlin Wall was demolished, one of the reunified Germany's most problematic ethnic
groups, in terms of cultural and economic assimilation, was the former East Germans

D)when the Berlin Wall was demolished, one of the most problematic ethnic groups in the reunified
Germany, in terms of cultural and economic assimilation, has been the former East Germans

E)after the Berlin Wall had been demolished, one of the most problematic ethnic groups in the
reunified Germany, in both terms of cultural and economic assimilation, have been the former East
Germans

Its quite clear, its C or D. So my question is which one and why?


OA:
SPOILER: D

Descending approximately 4,000 years ago from the African wildcat, it has been an exceedingly
short time for the domestic cat with respect to genetic evolution and it scarcely seems sufficient to
150

Adhamjons SC notes

UZBEKISTAN

allow the marked physical changes that transformed the animal.

A)Descending approximately 4,000 years ago from the African wildcat, it has been an exceedingly
short time for the domestic cat with respect to genetic evolution and it scarcely seems sufficient to
allow the marked physical changes that transformed the animal.

B)The domestic cat descended from the African wildcat approximately 4,000 years ago, which is an
exceedingly short time for the domestic cat's genetic evolution and scarcely sufficient for the
marked physical changes that transformed the animal.

C)Descending from the African wildcat approximately 4,000 years ago, the domestic cat has had an
exceedingly short time for its genetic evolution and has been scarcely sufficient for the marked
physical changes in the animal.

D)Having descended from the African wildcat approximately 4,000 years ago, the domestic cat has
had an exceedingly short time for its genetic evolution that has scarcely been sufficient for the
marked physical changes that transformed the animal.

E) The domestic cat descended from the African wildcat approximately 4,000 years ago, an
exceedingly recent divergence with respect to genetic evolution and one which scarcely seems
sufficient to allow the marked physical changes in the animal.

OA:
SPOILER: E
Why?

EXPLANATION BY GMATLOVE:
PROBLEM 1:

Since 1989, after the Berlin Wall had been demolished, one of the most problematic ethnic groups in the
reunified Germany, in cultural and economic assimilation terms, were the former East Germans, who have
had to acclimate to an entirely different political system.

C has the following problems:


151

Adhamjons SC notes

UZBEKISTAN

How many reunified Germanys do we know to talk about one of them?

C)when the Berlin Wall was demolished, one of the reunified Germany's most problematic ethnic groups,
in terms of cultural and economic assimilation, was the former East Germans

D)when the Berlin Wall was demolished, one of the most problematic ethnic groups in the reunified
Germany, in terms of cultural and economic assimilation, has been the former East Germans

My choice is D; because (a) concise and (b) there are no misplaced modifiers

In Problem 2; I have marked the problematic areas

Descending approximately 4,000 years ago from the African wildcat, it has been an exceedingly short
time for the domestic cat with respect to genetic evolution and it scarcely seems sufficient to allow the
marked physical changes that transformed the animal.

A)Descending approximately 4,000 years ago from the African wildcat, it (No antecedent for this
pronoun) has been an exceedingly short time for the domestic cat with respect to genetic evolution and it
scarcely seems sufficient to allow the marked physical changes that transformed the animal.

The tense is a problem in B. 'descended' past perfect indicates that the act was completed 4000 years ago.
While evolution is a continuous process which spans over 4000 years in domestic cats.

B)The domestic cat descended from the African wildcat approximately 4,000 years ago, which is an
exceedingly short time for the domestic cat's genetic evolution and scarcely sufficient for the marked
physical changes that transformed the animal.

Same as B + change of voice


C)Descending from the African wildcat approximately 4,000 years ago, the domestic cat has had an
exceedingly short time for its genetic evolution and (what) has been scarcely sufficient for the marked
physical changes in the animal.

In D; It is not clear if the time or the evolution is in question? Has the genetic evolution been insufficient
152

Adhamjons SC notes

UZBEKISTAN

or the cat had insufficient time to evolve?

D)Having descended from the African wildcat approximately 4,000 years ago, the domestic cat has had an
exceedingly short time for its genetic evolution that has scarcely been sufficient for the marked physical
changes that transformed the animal.

best of the lot! E

E) The domestic cat descended from the African wildcat approximately 4,000 years ago, an exceedingly
recent divergence with respect to genetic evolution and one which scarcely seems sufficient to allow the
marked physical changes in the animal.

EXPLANATION BY BOB:

I generally prefer not to get involved in discussions of bogus questions, but I have been asked to comment
on question 2. All the choices are bad.

A) Descending approximately 4,000 years ago from the African wildcat, it has been an exceedingly short
time for the domestic cat with respect to genetic evolution and it scarcely seems sufficient to allow the
marked physical changes that transformed the animal.
Introductory modifying phrase "descending..." requires that "domestic cat" be the subject.

B) The domestic cat descended from the African wildcat approximately 4,000 years ago, which is an
exceedingly short time for the domestic cat's genetic evolution and scarcely sufficient for the marked
physical changes that transformed the animal.
"Which" seems to be referring to "4,000 years ago," an adverb phrase. Should refer to a noun.

C) Descending from the African wildcat approximately 4,000 years ago, the domestic cat has had an
exceedingly short time for its genetic evolution and has been scarcely sufficient for the marked physical
changes in the animal.
This sentence says that "the domestic cat... has been scarcely sufficient." Nonsense.

D) Having descended from the African wildcat approximately 4,000 years ago, the domestic cat has had
153

Adhamjons SC notes

UZBEKISTAN

an exceedingly short time for its genetic evolution that has scarcely been sufficient for the marked
physical changes that transformed the animal.
"That has scarcely been sufficient" is placed so as to modify (illogically) "evolution."

E) The domestic cat descended from the African wildcat approximately 4,000 years ago, an exceedingly
recent divergence with respect to genetic evolution and one which scarcely seems sufficient to allow the
marked physical changes in the animal.

Horrible sentence. "Exceedingly recent divergence with respect to genetic evolution..."?!?! That's
the kind of inflated and imprecise verbiage I would expect of a bureaucrat, not of a correct response
in GMAT Sentence Correction. Does that really mean anything? And then "one which scarcely
seems sufficient..." One what? Divergence? Furthermore, "which" is used here to introduce a
restrictive clause, something that happens a lot in the real world but not in correct responses to
GMAT Sentence Correction items.

154

You might also like